Текст
                    Пойа Дж. и Килпатрик Д.
Сборник задач по математике Стэнфордского университета: с
подсказками и решениями. - М.: НО Научный Фонд «Первая Иссле-
довательская Лаборатория имени академика В.А. Мельникова», 2002.
-96 с.
ISBN 5-901171-04-7
Книга выдающегося американского математика и педагога Дж. Пойи, автора
таких известных книг, как «Математическое открытие», «Как решать задачу»,
«Математика и правдоподобные рассуждения», и Д. Килпатрика, содержит
оригинальные задачи повышенной трудности по элементарной математике,
которые предлагались американским школьникам на протяжении ряда лет на
конкурсах в Стэнфордском.университете США. \
Все задачи снабжены подсказками к решениям и подробными решениями.
Для широких кругов читателей, учителей и всех любителей математики.


Содержание Часть первая. Введение 4 Часть вторая. Задачи 9 Часть третья. Подсказки к решению задач 34 Часть четвертая. Решения задач 48 Литература 90
Часть первая Введение В течение двадцати лет, с 1946 по 1965 год, Департамент Математики Стэнфордского университета проводил конкурс- ные экзамены для выпускников старших классов средней шко- лы. Непосредственной и принципиальной целью экзаменов бы- ло определить среди выпускников средней школы каждого го- да особенно способных и одаренных студентов и привлечь их в Стэнфордский университет. Более широкой целью было сти- мулировать интерес к математике среди учащихся старших классов средних школ и учителей. Экзамены проводились по образцу конкурсов Этвеша [23], проводимых в Венгрии в 1894 г., который, в свою очередь, был принят для конкурсов в Англии и во Франции. Габор Же- го, председатель Стэнфордского Департамента Математики в 1946г. и победитель конкурса Этвеша в 1912 году, явился ини- циатором Стэнфордских экзаменов. Экзамены были учреждены в надежде, что раннее проявле- ние математических способностей является определенным по- казателем исключительного интеллекта и соответственно ин- теллектуального превосходства в любой другой области прило- жения усилий. Более того, математические способности могут
быть определены в сравнительно раннем возрасте; потому что они доказываются "не столько большим количеством накоп- ленных знаний, сколько оригинальностью мышления, проявля- ющейся в игровой схватке с трудными, хотя и элементарными задачами" [2, с. 406]. Как заметил Бук [1] несколько лет назад в обзоре математи- ческих конкурсов, экзамен может быть предназначен, вообще говоря, или для проверки знаний, или для проверки способ- ностей. Конкурсные экзамены по математике в Стэнфордском университете были последнего типа. Ударение делалось на оригинальность и интуицию более, нижели на шаблонные навыки... Типичный вопрос мог требо- вать специальных знаний, но мог требовать и при- менения необычных путей, требующих высокого уровня изобретательности. Вопрос может в дейст- вительности вводить определенное понятие, кото- рое является совершенно незнакомым учащемуся. Короче говоря, от учащегося требуется продемон- стрировать свои исследовательские способности [1, с. 204 - 205]. Первый Стэнфордский экзамен в 1946 году был проведен в 60 Калифорнийских школах в старших классах с 322 участ- никами. Победителю была присуждена одногодичная стипен- дия Стэнфордского университета; похвальный отзыв и книга по математике были даны трем другим участникам. В 1953 году экзамен был распространен за пределы Калифорнии - в Аризоне, Орегоне и Вашингтоне; количество стипендий было увеличено до двух, а количество присужденных похвальных отзывов и книг было увеличено до десяти и более. С 1958 по 1962 г. соучередителем экзаменов была компания Сильвания Электрик Продактс Инкорпорейтид. Последний экзамен в 1965 году был проведен среди 1200 участников в 151 центре в Кали- форнии, Аризоне, Айдахо, Монтане, Неваде, Орегоне и Вашинг- тоне. Денежные премии в 500, 250 и 250 долларов США были присуждены трем победителям; почетные отзывы и математи- ческие книги были вручены восемнадцати участникам. Экза- мены были прерваны после 1965 года главным образом потому,
что СтэнфордскиЙ Департамент Математики переключил свои интересы на обучение окончивших школу. Уведомления об экзамене посылались каждый год всем го- сударственным и частным .школам, в которых должны были проводиться экзамены. Большинство школ были назначены центрами, учащиеся из других школ были свободны в своем выборе где экзаменоваться согласно удобству их расположения для них. Экзамены проводились учителями и школьным персоналом по субботам, днем, в марте или апреле. Участникам давалось три часа на решение от трех до пяти задач. Была дана следу- ющая инструкция: Нельзя использовать какие-либо книги или тетра- ди. Вы можете не сделать все задачи за три часа, но все, что вы сделаете, следует внимательно обду- мать. Нельзя задавать вопросов, касающихся реше- ния задач. Решения задач должны быть написаны ясно, кратко и исчерпывающе. Работы читались в два этапа: на первом этапе они читались бригадой аспирантов в Департаменте Математики, включая, когда это было возможно, аспирантов, которые имели опыт ра- боты в старших классах средней школы. Каждой бригаде из двух студентов была поручена задача прочитать как можно больше работ. Работы, содержащие установленный минимум хороших решений (например, одно и половину или два из че- тырех) или некоторые оригинальные черты, направлялись на проверку второго этапа. На втором этапе каждая работа, ко- торая выдержала первый отсев, прочитывалась, по крайней мере, одним из членов профессорско-преподавательского со- става департамента. Работы, претендующие на победителя, прочитывались всеми участвовавшими членами профессорско- преподавательского состава. Чтобы упростить выбор победителя, задачи были придума- ны так, что только очень немногие участники смогли бы ре- шить все из них. С другой стороны, для того, чтобы избежать слишком большого разочарования, первая задача была обычно более доступна по уровню нежели другие, особенно в послед-
ние годы, так что многие участники были способны решить ее. Хотя математическая сущность задач не выходила за пре- делы школьной программы, задачи относились к типу редко встречающихся в учебниках. Целью такой задачи являлась не только проверка оригинальности учащегося, но также обога- щение программы по математике средней школы посредством предложения некоторых новых направлений для работы уча- щихся и учителей. Задачи включали следующие типы: A) "предложение и доказательство", в которых сначала делается предположение, а затем доказывается математический факт; B) "проверка следствий", в которых проверяется следствие из общих утверждений; C) "вы можете сделать ложное предпо- ложение", в котором весьма вероятное предположение явля- ется неверным; D) "небольшая теория", в которой последова- тельность подзадач иллюстрирует теоретическое построение; E) "отвлекающий маневр", в котором очевидное соотношение, оказывается, не имеет никакого отношения к решению [9, 19, с. 160 - 161, пример 1; 21, с. 139, пример 14.23]. Задачи были того же уровня, что и в книгах Как решать за- дачу [17], Математика и правдоподобные рассуждения [18, 19] и Математическое открытие [20, 21]. Действительно, мно- гие из задач и обычно с решениями, появляются в одной или нескольких из этих книг. Большинство задач опубликовано в журналах. Задачи и список победителей каждого экзамена с 1946 по 1953 г. (кроме 1952 г.) были опубликованы в American Mathematical Monthly [2 - 8], а полный набор задач вместе с Введением, приведенным здесь, появился в июне - июле 1973 г. в [24]. Статьи, содер- жащие задачи, решения, комментарии и список победителей с 1953 по 1961 г. (кроме 1959 г.), были опубликованы в California Mathematics Counsil Bulletin [9 - 16]. Полный набор задач вместе с подсказками и решениями всех задач никогда не публиковался. Часть вторая содержит полный набор задач, предлагавших- ся на конкурсных экзаменах по математике в Стэнфордском университете. Они пронумерованы последовательно, с указа- нием года и номера задачи, как например: 46Л означает задачу
8 1 на экзамене 1946 года1. Часть третья содержит подсказки к решению каждой зада- чи, которые пронумерованы в соответствии с номерами задач во второй части. Стиль подсказок подобен подсказкам в час- ти IV книги Как решать задачу [17] и большинство подсказок используют один или более наводящих вопросов и предположе- ний. Четвертая часть содержит решения всех задач (иногда два решения); нумерация - как прежде. Решения намечают в об- щих чертах используемые подходы, но некоторые детали, ко- нечно, оставляют читателю. Некоторые решения заканчива- ются указанием взаимосвязей с другими проблемами или об- общениями. Многие подсказки к решениям и решения задач появились в результате дискуссии на семинарах по решению задач, кото- рые проводились в Стэнфордском университете и Учительском коллеже. Задачи использовались для иллюстрации техники проблемных решений с первокурсниками, будущими учителя- ми и учителями, имеющими опыт. Учителя и учителя учите- лей могут найти некоторые полезные предложения по вопросу, как использовать задачи в Математическом Открытии [20, с. 209 - 212]. Ссылки на статьи и книги, в которых та или иная задача появилась впервые (с решением или без), приводятся в квад- ратных скобках, с соответствующими решениями в четвертой части. 1В данном издании принята сквозная нумерация, в которой указыва- ется номер группы задач и номер задачи в группе, например: 2.3 означает задачу 3 во второй группе.
Часть вторая Задачи Задачи 1 1.1. В теннисном турнире принимают участие In участни- ков. В первом круге турнира каждый участник играет только один раз, так что оказывается сыгранными п игр парами игро- ков. Показать, что количество пар в первом круге может быть составлено в точности 1хЗх5х7х9х---хBп-1) различными способами. 1.2. В тетраэдре (необязательно правильном) два противо- положных ребра имеют одинаковую длину а и они перпендику- лярны друг другу. Кроме того, каждое из них перпендикуляр- но линии длиной 6, которая соединяет их середины. Выразить объем тетраэдра через а и 6 и обосновать полученное выраже- ние. 1.3. Рассмотреть следующие четыре утверждения, которые необязательно являются истинными:
10 I. Если многоугольник, вписанный в окружность, являет- ся равносторонним, то он также является и равноуголь- ным. И. Если многоугольник, вписанный в окружность, являет- ся равноугольным, то он также является и равносторон- ним. III. Если многоугольник, описанный около окружности, яв- ляется равносторонним, то он является также и равно- угольным. IV. Если многоугольник, описанный около окружности, яв- ляется равноугольным, то он является также и равно- сторонним. (A) Установите, какие из четырех утверждений являются истинными, а какие ложными, давая доказательство вашего утверждения в каждом случае. (B) Если вместо многоугольников общего вида мы рассмот- рим только четырехугольники, какие из четырех утверждений истинны, а какие ложные? А если мы рассматриваем только пятиугольники? При ответе на (В) вы можете строить догадки, но докажите это как можно полнее и ясно отделите что доказано, а что не доказано.
11 Задачи 2 2.1. Чтобы пронумеровать страницы объемистого тома, печатник использовал 1890 цифр. Сколько страниц содержит том? 2.2. Среди дедушкиных бумаг был обнаружен счет: 72 индюшки -67,9- долларов. Первая и последняя цифры числа, которое, очевидно, представ- ляло собой общую цену этих птиц, заменены здесь тире, по- скольку они стерлись и стали неразборчивыми. Каковы две стершиеся цифры, и сколько стоил один индюк? 2.3. Определить га, при которых уравнение относительно х z4 - (Зш + 2)х2 + ш2 = 0 имеет четыре действительных корня, являющиеся членами ариф- метической прогрессии. 2.4. Пусть а, /3 и у являются углами треугольника. Пока- зать, что оч C у sin a + sin C + sin 7 = 4 cos — cos — cos —, ? Z* Zi sin 2a + sin 2/? + sin 2y = 4 sin a sin /3 sin 7 и sin 4a + sin 4/3 + sin 47 = -4 sin 2a sin 2C sin 27.
12 Задачи 3 3.1. Рассмотрим таблицу: 1 = 1, 2 + 3 + 4 =1 + 8, 5 + 6 + 7 + 8 + 9 =8+27, 10+11 + 12+13 + 14 + 15+16 = 27 + 64. Установите общий закон, заложенный в этом примере, вы- разите его математически и докажите. 3.2. Три числа являются членами арифметической прогрес- сии, три других числа - членами геометрической прогрессии. Складывая соответствующие члены двух прогрессий последо- вательно, мы получим 85, 76 и 84 соответственно, а складывая все три члена арифметической прогрессии мы получим 126. Найдите члены обеих прогрессий. 3.3. С вершины горы вы видите две точки А и В на плос- кости. Линии зрения, направленные на эти точки, составляют угол 7- Наклон первой линии зрения к горизонтальной плос- кости равен tv, а второй линии C. Известно, что точки А и В находятся на одном уровне и что расстояние между ними равно с. Выразите высоту х вершины над общим уровнем точек А и В через величины углов а, /3, 7 и расстояние с. 3.4. Первая сфера имеет радиус г\. Около этой сферы опи- сан правильный тетраэдр. Около этого тетраэдра описана вто- рая сфера с радиусом Г2. Около этой второй сферы описан куб. Около этого куба описана третья сфера с радиусом г3. Найти отношение г\ : r<i : гз (которое должно быть, соглас- но Кеплеру, отношением средних расстояний от планет Марс, Юпитер и Сатурн до Солнца, но которое, в действительности, несколько отличается от истинного отношения).
13 Задачи 4 4.1. Доказать, что нет такого числа в последовательности 11,111,1111,11111,..., которое является квадратом целого числа. 4.2. Три стороны треугольника имеют длину /, т и п со- ответственно. Числа /, т и п положительные и целые, I < т < п. (A) Положите п = 9 и найдите количество различных тре- угольиков описанного вида. (B) Возьмите различные значения п и найдите общую за- кономерность. 4.3. (А) Докажите следующую теорему: точка лежит внут- ри равностороннего треугольника и расположена на расстоянии ж, у и z от трех его сторон соответственно; h - высота треуголь- ника. Тогда х + у + z — h. (B) Установите и докажите аналогичную теорему в стерео- метрии относительно расстояний от внутренней точки до че- тырех граней правильного тетраэдра. (C) Обобщите обе теоремы так, чтобы они были бы примени- мы к любой точке в плоскости или пространстве соответствен- но (а не только к точкам внутри треугольника или тетраэдра). Дайте четкую и ясную формулировку утверждения и если у вас будет время, также и доказательство.
14 Задачи 5 5.1. Рассмотрим равенства 1 = 1, 1-4 = -A + 2), 1-4 + 9 = 1 + 2 + 3, 1-4 + 9-16 = -A + 2 + 3 + 4). Установите общий закон в данном примере, выразите его математически и докажите. 5.2. Дан квадрат. Найти геометрическое место точек, из которых квадрат виден под углом (А) 90°, (В) 45°. (Пусть Р является точкой вне квадрата, но в той же плоскости. Наи- меньший угол с вершиной Р, содержащий в себе квадрат, яв- ляется углом, под которым квадрат виден иэ точки Р). На- бросайте отчетливо оба геометрических места точек, дайте их описание и докажите. 5.3. Назовем осью тела прямую линию, соединяющую две точки поверхности тела, и такую, что тело, повернутое вокруг этой линии на угол, который больше 0° и меньше 360°, совпа- дает с самим собой. Куб имеет 13 различных осей, которые принадлежат трем различным классам. Ясно опишите расположение этих осей, найдите угол поворота, соответствующий каждой оси. При- нимая, что ребро куба имеет единичную длину, вычислите среднее арифметическое длин 13 осей. Не используя вычис- лительных таблиц, проведите вычисления до двух десятичных знаков после запятой.
15 Задачи 6 6.1. Периметр прямоугольного треугольника равен 60 дюй- мам, а длина высоты, перпендикулярной гипотенузе, равна 12 дюймам. Найдите стороны треугольника. 6.2. Четырехугольник разрезан на четыре треугольника его двумя диагоналями. Назовем два из этих треугольников "противоположными" если они имеют общую вершину и не имеют общих сторон. Доказать следующие утверждения: (A) Произведение площадей двух противоположных треуголь- ников равно произведению площадей двух других противопо- ложных треугольников. (B) Четырехугольник является трапецией тогда и только тогда, когда имеются два противоположных треугольника, рав- ных по площадям. (C) Четырехугольник является параллелограммом тогда и только тогда, когда все четыре треугольника равны по площа- дям. 6.3. Рассмотрим прямой усеченный круговой конус. Плос- кость, которая параллельна нижнему и верхнему основаниям усеченного конуса и расположена на равных расстояниях от обоих оснований, пересекает усеченный конус в "срединном круге". Усеченный конус и цилиндр имеют одинаковую высо- ту, и срединный круг конуса является основанием цилиндра. Какое из этих двух тел имеет больший объем: усеченный конус или цилиндр? Докажите свой ответ! (Возможным доказательством является алгебраическое: Вы- разите оба объема в подходящих переменных и преобразуйте их разность так, что знак этой разности становится очевид- ным).
16 Задачи 7 7.1. Докажите утверждение: если сторона треугольника меньше, чем среднее (среднее арифметическое) двух других сторон, то ее противоположный угол меньше, чем среднее двух других углов. 7.2. Рассмотрим прямую усеченную пирамиду с квадрат- ным основанием. Назовем "срединным сечением" пересече- ние пирамиды плоскостью, параллельной основанию и верхней плоскости пирамиды и расположенной на одинаковых рассто- яниях от обоих. Назовем "срединным прямоугольником" пря- моугольник, у которого одна сторона равна стороне основания, а другая сторона равна стороне верхнего основания. Четыре ваших друга согласны, что объем усеченной пира- миды равен высоте, умноженной на некоторую площадь, но они не согласны между собой относительно этой площади и выска- зывают следующие четыре предположения о том, что эта пло- щадь является: I. средним сечением; II. средней величиной нижнего и верхнего оснований; III. средней величиной нижнего основания, верхнего осно- вания и срединного сечения; IV. средней величиной нижнего основания, верхнего осно- вания и срединного прямоугольника. Пусть h является высотой усеченной пирамиды, а - сторо- ной ее основания и 6 - стороной ее верхнего основания. Выразите каждое из четырех предположений в математи- ческой форме, решите какое из предположений правильное и докажите свой ответ. 7.3. Доказать, что единственным решением уравнения х2 + у2 + z2 = 2xyz в целых числах х, у и z является х = у = z = 0.
17 Задачи 8 8.1. У Боба 10 карманов и 44 серебряных доллара. Он хочет распределить доллары по своим карманам так, чтобы в каждом кармане содержалось различное количество долларов. (A) Сможет ли он сделать это? (B) Обобщите проблему, полагая количество карманов рав- ным и количество долларов равным п. Проблема наиболее интересна, когда ТЬ — 2 Почему? 8.2. Заметьте, что значение 12 3 2! ' 3! 4! (гс+1)! равно 1/2, 5/6, 23/24 для п = 1, 2, 3 соответственно. Опреде- лите общую закономерность (давая п большие значения, если необходимо) и докажите ваш закон. 8.3. Найти х, у, и и v, удовлетворяющие системе четырех уравнений x + 7y + 3v + 5u = 16, 8x + 4y + Qv+2u = -16, 2ж + 6у + 4и + 8w = 16, bx + 3y + 7v+ и = -16. (Это может показаться долгим и скучным, но поищите путь покороче). 8.4. Четыре точки G*, Я, V и U являются (в указанном порядке) четырьмя вершинами четырехугольника. Геодезист хочет определить длину UV = х. Он знает длину GH = I и измеряет
18 (A) Выразите х через а, /3, 7» ^ и ^ (B) Найдите путь проверить правильность результата. (C) Если у вас есть ясный план как сделать (А) охаракте- ризуйте его одним коротким предложением.
19 Задачи 9 9.1. Рассмотрим равенства 1 = 1, 3 + 5 =8, 7 + 9 + 11 = 27, 13+15 + 17 + 19 = 64, 21 + 23 + 25 + 27+29 = 125. Установите общий закон в данном примере, выразите его ма- тематически и докажите. 9.2. Сторона правильного шестиугольника имеет длину п (п является целым числом). С помощью равноотстоящих, па- раллельных его сторонам прямых, шестиугольник разбит на Т равносторонних треугольников, каждый из которых имеет стороны длиной 1. Пусть V означает количество вершин, по- явившихся при разбиении, a L - количество граничных линий длиной 1. (Граничная линия принадлежит одному или двум треугольникам, вершина - двум или более треугольникам). Когда п = 1 (простейший случай), Т = 6, V = 7, L = 12. Рассмотреть общий случай и выразить Т, V и L через п. (Догадка - хорошо, но доказательство - лучше). 9.3. Покажите, что невозможно определить (действитель- ные или комплексные) числа а, 6, с, Л, Б и С такие, что урав- нение х2 + у2 + z2 = (ax + by + cz)(Ax + By + Cz) выполняется тождественно для независимых переменных ж, у и z.
20 Задачи 10 10.1. Боб хочет иметь кусок земли идеально ровный, кото- рый ограничен четырьмя линиями. Две граничные линии идут строго на север-юг, две другие - строго на восток-запад, и каж- дая граничная линия измеряется точно 100 футами. Сможет Боб купить такой кусок земли в США? Обоснуйте! 10.2. (А) Определите три числа р, q и г такие, что уравне- ние х4 + 4х3 - 2х2 - \2х + 9 = (рх2 + qx + гJ выполняется тождественно для переменной х. (В) Эта задача требует точного извлечения квадратного кор- ня данного многочлена четвертой степени, которое может быть возможным в данном случае, хотя обычно это невозможно. По- чему невозможно? 10.3. Боб, Петер и Пауль путешествуют вместе. Петер и Пауль - хорошие пешеходы; каждый проходит р миль в час. У Боба - больные ноги и он едет на маленькой машине, в которой могут ехать два человека, но не три; машина покрывает с миль в час. Трое друзей приняли следующий план: они стартуют вместе, Пауль едет в машине вместе с Бобом, а Петер идет пешком. После некоторого промежутка времени, Боб высажи- вает Пауля, который идет пешком; Боб поворачивает, чтобы забрать Петера и затем Боб и Петер едут в машине, пока не догонят Пауля. В этом месте они меняются: Пауль едет, а Пе- тер идет пешком так же, как они начинали, и вся процедура повторяется столько, сколько это необходимо. (A) Как далеко (сколько миль) продвигается компания за час. (B) В течение какого промежутка от времени путешествия машина везет только одного человека? (C) Проверьте предельные случаи р = 0 и р = с. 10.4. Вершина пирамиды, расположенная против ее осно- вания называется верхушкой.
21 (A) Назовем пирамиду "равнобедренной", если ее верхушка расположена на одинаковом расстоянии от всех вершин осно- вания. Приняв это определение, доказать, что основание рав- нобедренной пирамиды вписывается в окружность, центр ко- торой является основанием высоты пирамиды. (B) Теперь назовем пирамиду равнобедренной, если ее вер- хушка расположена на одинаковом (по перпендикулярам) рас- стоянии от всех сторон основания. Приняв это определение (отличное от вышеприведенного), доказать, что основание рав- нобедренной пирамиды описывается около окружности, центр которой является основанием высоты пирамиды.
22 Задачи 11 11.1. Дан правильный шестиугольник и точка в его плос- кости. Провести прямую линию через данную точку, которая делит данный шестиугольник на две части равной площади. 11.2. Я утверждаю, что вы можете заплатить 50 центов в точности 50-ю различными способами. (Способ зависит от то- го, как много монет каждого вида - центов, пятицентовых мо- нет, десятицентовых монет, четверть- и полдолларовых монет вы используете). Сколькими способами вы можете заплатить 25 центов? Прав ли я относительно 50 центов? Подтвердите ваш ответ как можно более четко и ясно. 11.3. Постройте шестиугольник путем добавления к про- извольно заданному треугольнику Л трех внешних равнобед- ренных треугольников, каждый из которых имеет угол 120° против той стороны Л, которая образует его основание. Пока- зать, что три вершины шестиугольника, которые не являются вершинами данного Л, являются вершинами равностороннего треугольника. (Достаточно выразить только одну сторону s упомянутого выше равностороннего треугольника через сто- роны а, 6 и с треугольника Л, доказав, что это выражение для s симметрично относительно а, 6 и с). 11.4. Десять человек сидят вокруг стола. Сумма в десять долларов должна быть распределена среди них по правилу, со- гласно которому каждый получает одну половину от суммы, которую его два соседа получают вместе. Есть ли только один путь, чтобы распределить деньги таким образом? Докажите ваш ответ.
23 Задачи 12 12.1. Коллекция марок Боба состоит из трех альбомов. Две десятых его марок находятся в первом альбоме, несколько седьмых - во втором альбоме и 303 марки в третьем альбоме. Сколько марок у Боба? (Является ли условие достаточным, чтобы определить неизвестное?) 12.2. Назовем вершину тетраэдра трехпрямоугольной, ес- ли три ребра, начинающиеся от нее, перпендикулярны друг к другу. Имея площади А, В и С трех граней, примыкающих к трехпрямоугольной вершине тетраэдра, определить площадь D четвертой грани противоположной этой вершине. (Какую задачу в планиметрии вы бы отнесли к аналогичной?) 12.3. Разбейте данный треугольник тремя прямыми се- кущими на семь частей, четыре из которых являются тре- угольниками (а оставшиеся три пятиугольниками). Одна из треугольных частей образована тремя секущими, каждая из трех других треугольных частей образована некоторой сторо- ной данного треугольника и двумя секущими. (A) Выберите три прямые секущие так, чтобы четыре тре- угольных части оказались конгруэнтными. Опишите ваш выбор и нарисуйте ясный чертеж. (B) Какая часть площади данного треугольника является площадью треугольной части в рассечении, которое вы выбрали? (Может быть полезным рассмотреть сначала частный вид данного треугольника, для которого решение является особен- но простым).
24 Задачи 13 13.1. Сколько лет капитану, сколько у него детей и какова длина его корабля? Дано число 32118, являющееся произведе- нием трех искомых чисел (целых). Длина корабля дана в футах (и равна нескольким футам), капитан имеет двоих сыновей и дочерей, ему больше лет, чем детям, но ему еще не сто лет. (Обоснуйте ответ). 13.2. Определите ж, у, и и v удовлетворяющих системе четырех уравнений х + у + и - 4, у + и + v — -5, и + v + х — О, v + х + у = -8. (Это может показаться долгим и скучным: поищите укорочен- ный путь). 13.3. "В любом треугольнике сумма трех... больше, чем полупериметр". Восстановите пропущенное (отмеченное точками) последо- вательно выбирая: I. высоты; П. медианы; III. биссектрисы (углов). Вы получаете три различных утверждения. Проверьте каж- дое утверждение: истинно оно или ложно? Докажите ваш от- вет! 13.4. Обратите внимание, что значение 1!1 + 2!2 + 3!3+••¦ + «!» равно 1, 5, 23, 119 для п = 1, 2, 3, 4 соответственно. Устано- вите общий закон (путем задания дальнейших значений если необходимо) и докажите ваше утверждение.
25 Задачи 14 14Л. Эл и Билл живут на противоположных концах одной улицы. Элу необходимо доставить посылку к дому Билла, а Биллу - к дому Эла. Они вышли в один и тот же момент, каж- дый шел с постоянной скоростью и возвратился домой сразу же после оставления посылки в месте назначения. Они встре- тились в первый раз на расстоянии а ярдов от дома Эла и во второй раз на расстоянии 6 ярдов от дома Билла. (A) Какова длина улицы? (B) Если а = 300 и 6 = 400, кто шел быстрее? 14.2. Монеты (равной окружности) расположены в регуляр- ную структуру по всему очень большому столу (бесконечная плоскость). Рассмотрим две структуры. В первой структуре каждая монета касается четырех дру- гих монет, и прямые линии, соединяющие центры контакти- рующих монет, рассекают плоскость на равные квадраты. Во второй структуре каждая монета касается шести дру- гих монет и прямые линии, соединяющие центры соприкасаю- щихся монет, разбивают плоскость на равные равносторонние треугольники. Вычислите процент плоскости, покрытой монетами (круга- ми) для каждой структуры. 14.3. Доказать: если п является целым числом, большим 1, то пп~1 — 1 делится на (п — IJ без остатка. 14.4. Постройте (внешний) квадрат на каждой стороне (про- извольного) треугольника. Те 6 вершин этих трех квадратов, которые не совпадают с вершинами треугольника, образуют шестиугольник. Три стороны этого шестиугольника равны, конечно, соответствующим сторонам треугольника. Показать, что каждая из оставшихся трех сторон равна удвоенной меди- ане треугольника.
26 Задачи 15 15.1. Новые шариковые ручки стоили 50 центов в магази- не напротив школы, нЬ покупателей этих ручек было немного. Когда, однако, магазин понизил цену, весь оставшийся запас ручек был продан за 31,93 доллара. Насколько была сниже- на цена? (Является ли условие достаточным для определения неизвестного?) 15.2. Точка Р так расположена внутри прямоугольника, что расстояние от точки Р до угла прямоугольника составляет 5 ярдов, от противоположного угла 14 ярдов, а от третьего угла 10 ярдов. Каково расстояние от точки Р до четвертого угла? 15.3. Докажите тождество a a a sin a cos — cos — cos — = 2 4 8 8sin| и обобщите его. 15.4. Из двенадцати конгруэнтных равносторонних тре- угольников восемь являются гранями правильного октаэдра, а четыре - гранями правильного тетраэдра. Найти отношение объема октаэдра к объему тетраэдра.
27 Задачи 16 16.1. Решите следующую систему трех уравнений с неиз- вестными я, у и z: 5732а:+ 2134?/+ 21342 = 7866, 2134а; + 5732?/ + 2Шг = 670, 2134а;+ 2134?/+ 57322 = 11464. 16.2. Был очень жаркий день, и четыре пары выпили вмес- те 44 бутылки кока-колы. Анна выпила 2, Бетти 3, Кэрол 4 и Дороти 5 бутылок. М-р Браун выпил столько же бутылок, сколько и его жена, но каждый из других мужчин выпил боль- ше чем его жена: м-р Грин вдвое, м-р Вайт в три раза и м-р Смит в четыре раза. Назовите жен этих мужчин. (Докажите ваш ответ). 16.3. Решите следующую систему трех уравнений с неиз- вестными х, у и z (a, b и с заданы) х2у2 + х2 z2 = axyz, y2z2+y2x2 - bxyz, 9 1. 9 9 z~x~-\-z~y = cxyz. 16.4. Пирамида называется ''правильной", если ее осно- вание является правильным многоугольником, а основание ее высоты находится в центре основания пирамиды. Правильная пирамида имеет шестиугольное основание, площадь которого является одной четвертой от общей площади поверхности S пи- рамиды. Высота пирамиды равна h. Выразите S через h.
28 Задачи 17 17.1. Решить систему уравнений 2х2 - 4ху + Зу2 = 36, 3z2 - 4ху + 2у2 = 36. (Одно решение установить легко, но от вас требуется найти все решения. Знание аналитической геометрии не требуется для решения этой задачи, но может помочь понять результат - как?) 17.2. Каждое из четырех чисел а, 6, с и d является поло- жительным и меньше единицы. Показать, что не все четыре произведения 4аA-6), 46A-с), 4c(l-rf), 4d(l-a) больше единицы. 17.3. На каждой стороне прямоугольного треугольника по- строен наружный квадрат (как это обычно делается для ил- люстрации теоремы Пифагора). Соедините вершину прямого угла треугольника с центром квадрата, построенного на гипо- тенузе, и соедините центры квадратов, построенных на двух других сторонах. Показать, что два отрезка прямых, получен- ных таким образом являются (A) перпендикулярными друг другу; (B) равной длины. 17.4. Пять ребер тетраэдра имеют одинаковую длину a, a шестое ребро имеет длину Ь. (A) Выразите радиус сферы, описанной около тетраэдра, че- рез а и Ь. (B) Как вы могли бы использовать результат (А) для прак- тического определения радиуса сферической поверхности (на- пример линзы)?^
29 Задачи 18 18.1. В прямоугольном треугольнике, с - длина гипотену- зы, а и 6 - длины двух других сторон, ad- диаметр вписанной окружности. Доказать, что а + b = с + d. 18.2. Показать, что выражение п2(п2-1)(п2-4) делится на 360 без остатка для п =1, 2, 3,.... 18.3. Найдите все решения системы трех уравнений с не- известными я, у и z х2+Ьу2+§z2+ S{yz + zx + xy) = 36, 6z2 + г/2 +5z2 + 8(yz+zx + xy) = 36, 5ж2+6г/2-(- z2 + 8(yz + zx + xy) = 36. (Одно решение найти легко). 18.4. Основание прямой призмы является правильным шес- тиугольником, а высота призмы равна диаметру окружности, вписанной в основание. Объем призмы равен объему правиль- ного октаэдра. Найти отношение площадей поверхностей этих двух тел. Обратите внимание, что два тела имеют одинаковое коли- чество граней, и одно из них является правильным телом, а другое - нет. Почему так?
30 Задачи 19 19.1. Торт имеет форму прямой призмы с квадратным ос- нованием; он имеет сахарную глазурь наверху торта и по его сторонам (то есть на четырех боковых гранях). Высота приз- мы равна 5/16 стороны основания. Разрежьте торт на 9 кусков так, чтобы каждый кусок имел одинаковое количество торта и глазури. Один из 9 кусков должен быть прямой призмой с квадратным основанием, с глазурью только на его верхней поверхности. Вычислите отношение его высоты к стороне его основания, и дайте четкое описание с эскизами всех 9 кусков. 19.2. Показать, что каждое число последовательности 49, 4489, 444889, 44448889,... является точным квадратом. 19.3. Если площадь треугольника является рациональной (т.е. измеряется рациональным числом), то существует че- тыре возможных случая: треугольник может иметь три или две рациональных стороны, или только одну, или ни одной ра- циональной стороны. Показать на примерах (предпочтительно простых), что все четыре случая на самом деле возможны. 19.4. Экзамены по трем предметам: алгебре, биологии и химии сдавали 41 студент. Следующая таблица показывает, сколько студентов провалились на каждом предмете, и их раз- личные комбинации: Предмет А Б X АБ АХ БХ АБХ Количество 12 5 8 2 6 3 1 провалившихся (К примеру, 5 студентов провалились по биологии, среди них 3 провалившихся как по биологии, так и по химии, и только один из этих 3 провалился по всем трем предметам). Сколько студентов сдали все три экзамена?
31 (Можете ли вы обдумать подходящую диаграмму, которая прояснила бы лежащую в основе идею?). 19.5. Пусть а, 6 и с означают длины сторон треугольника, а d - длину биссектрисы угла, противоположного стороне длиной с и оканчивающуюся на этой стороне. (A) Выразить d через а, 6 и с. (B) Проверьте полученное выражение самыми различными путями, которые можете придумать (с помощью частных слу- чаев, предельных случаев и т.д.)
32 Задачи 20 20.1. "Сколько у Вас детей и сколько им лет?" - спросил гость у учителя математики. "У меня три мальчика, - сказал м-р Смит. - Произведение их лет равно 72, а сумма их лет равна номеру улицы". Гость пошел посмотреть номер, вернулся и сказал: "Проб- лема не определена". "Да, это так, - сказал м-р Смит, - но я все еще надеюсь, что старший мальчик однажды выиграет конкурс Стэнфордского Университета". Назовите возраст мальчиков, излагая ваши соображения. 20.2. Для прямоугольного треугольника даны длина ги- потенузы с и площадь Л. На каждой стороне треугольника начерчен квадрат, внешний к треугольнику, и рассматрива- ется наименьшая выпуклая фигура, состоящая из трех квадра- тов (сформированная туго натянутой резиновой лентой вокруг них): это шестиугольник (который является неправильным, имеет по одной стороне общей с каждым квадратом, и одна из его оставшихся трех сторон, очевидно, имеет длину с). Определить площадь шестиугольника. 20.3. Пусть числа х, у и 1 измеряют длины трех сторон некоторого треугольника и предположим, что х < у < 1. Пусть точка (х, у) с координатами хну прямоугольной системы координат представляет треугольник на плоскости. Опишите четко и ясно набор этих точек на плоскости, который представляет (A) треугольники; (B) равнобедренные треугольники; (C) прямоугольные треугольники; (D) острые треугольники; (E) тупые треугольники.
33 Определите положение точек, представляющих и другие, за- служивающие внимания треугольные фигуры. 20.4. Найти остаток от деления полинома на полином ж3 — х.
34 Часть третья Подсказки к решению задач Подсказки 1 1.1. Не могли бы Вы вновь посмотреть на задачу! Пред- ставьте, что Вы один из участников. Сколькими способами Вы можете выбрать себе пару? Подумайте, как проблема может быть разбита на две части: (A) выбор Вашего противника и (B) образование пар между оставшимися участниками. 1.2. Посмотрите на неизвестные. Неизвестной являетсд объем тетраэдра, который может быть вычислен, когда осно- вание и высота заданы. Но ни основание, ни высота не заданы в этой задаче. Не могли бы Вы вообразить более доступную родственную задачу! (Не видите ли Вы более доступного тет- раэдра, который является кратной частью исходного тетраэд- ра?)
35 1.3. Доказательство любого из утверждений сведите к до- казательству равенства или линейных отрезков, или углов. Зна- ете ли Вы теорему или теоремы, используемые при доказатель- стве их равенства? Нарисуйте чертеж. Может Вам ввести вспомогательные элементы,, чтобы сделать возможным исноль- зование теорем? Подсказки 2 2.1. Вот Вам задача: если книга имеет точно 9 пронумеро- ванных страниц, сколько цифр использовал печатник? (Ответ: 9). Л вот Вам другая задача: если книга имеет точно 99 про- нумерованных страниц, сколько цифр использовал печатник в этом случае? 2.2. Вновь посмотрите на задачу. Какими могут быть две стершиеся цифры, если общая цена, выраженная в центах де- лится на 72? 2.3. Каковы условия! Четыре корня должны образовывать арифметическую прогрессию. Еще: уравнение имеет специ- фическую черту - оно содержит только четные степени не- известной х. Следовательно, если а является корнем, то —а также является корнем. 2.4. Знаете ли Вы подходящую теорему! Заметьте сход- ство между тремя тождествами, особенно в их левых частях. Если Вы установили одно тождество, как можно получить дру- гие два? Подсказки 3 3.1. Применение метода индукции требует наблюдатель- ности. Посмотрите на члены справа, начальные члены слева и члены в конце. Какова обшая закономерность?
36 3.2. Разделить различные части условия. Можете Вы за- писать их? Пусть a — d, а, а + d будут членами арифметической прогрессии, а bg~\ Ь, bg будут членами геометрической прогрессии. 3.3. Разделите различные части условия. Можете Вы за- писать их! Пусть а и b означают длины (неизвестных) линий зрения, а и C - их наклон к горизонтальной плоскости соот- ветственно. Мы можем отметить три чаети в условиях отно- сительно A) наклона линии а, B) наклона линии 6, C) треугольника со сторонами а, 6 и с. 3.4. Посмотрите на неизвестные. Имеются две неизвест- ные: отношение г\ : г^ и отношение г2 : гз- Подсказки 4 4.1. Вновь посмотрите на задачу. Проблема .которую надо решить: "найти полный квадрат выражения 1 + 10 + 102 И \- Ю^, где к - положительное целое число". Разделите различные части условия. Можете Вы записать их? Можно отметить две части в условии: A) s должна быть квадратом, B) s должна иметь требуемый вид. 4.2. Применение метода индукции требует наблюдатель- ности. Можете Вы найти методику подсчета треугольников для данного значения п? 4.3. Знаете ли Вы подходящую теорему? Высота равна h. Знаете ли Вы более простую теорему относительно высоты в треугольнике?
37 Подсказки 5 5.1. Применение метода индукции требует наблюдатель- ности. Рассмотрите переход от одного выражения к следую- щему. 5.2. Знаете ли Вы родственную задачу! Геометрическое место точек, из которых данный отрезок прямой линии виден под данным углом, состоит из двух дуг, оканчивающихся в крайних точках отрезка и симметричных друг относительно друга относительно отрезка. 5.3. Некоторые оси могут быть установлены просто осмот- ром, но являются ли все оси осями? Можете Вы доказать, что Ваш перечень осей исчерпывающий? Имеет ли Ваш перечень ясный принцип классификации? Подсказки 6 6.1. Разделите различные части условия. Можете Вы запи- сать их7. Можно отметить три части в условии относительно A) периметра, B) прямоугольного треугольника, C) высоты к гипотенузе. 6.2. Нарисуйте чертеж. Введите удобные обозначения. Как можно показать, что площади (или произведения площадей) равны? 6.3. Нарисуйте чертеж. Введите удобные обозначения. Подсказки 7 7.1. Что такое гипотеза? Что такое умозаключение? Пусть а, 6 и с означают стороны, а А, В и С - противоположные углы
38 соответственно. Тогда гипотезой является следующее неравен- ство Ь + с а < а умозаключением Посмотрите на умозаключение. Можете ли Вы переформули- ровать его? 7.2. Знаете ли Вы родственную задачу? Л более частную задачу? Что произойдет, если изменить данные в задаче? 7.3. Каковы условия? Сумма квадратов трех целых чисел должна быть удвоенным их произведением. Сумма должна быть четным числом. Подсказки 8 8.1. Если у Боба очень много долларов, у него, очевидно, не было бы трудностей при заполнении каждого из своих кар- манов различным количеством долларов. Можете ли Вы пере- формулировать задачу? Каково наименьшее количество долла- ров, которое может быть положено в 10 карманов так, чтобы никакие два различных кармана не содержали одинаковое их количество? 8.2. Узнаете ли Вы знаменатели 2, 6, 24? Знаете ли Вы родственную задачу? Аналогичную задачу? 8.3. Чтобы решить такую систему, мы вынуждены ком- бинировать уравнения каким-либо образом. Подыщите связи между уравнениями, которые могли бы привестм к особенно выгодной комбинании. 8.4. Ньютон однажды заметил, что в некоторой геометри- ческой задаче получается та же система независимо от того,
39 какие величины рассматриваются как исходные данные, а ка- кие - как неизвестные. Поэтому следует выбрать данные и неизвестные так, чтобы было легко вывести уравнение. Подсказки 9 9.1. Применение метода индукции требует наблюдатель- ности. Обратите внимание на правые части, начальные члены в левой части и конечные члены. Какова общая закономер- ность? 9.2. Нарисуйте чертеж. Он может помочь Вам установить закон индуктивно или может привести Вас к соотношениям между Г, V, L и п. 9.3. Возможно ли удовлетворить условию? Является ли условие достаточным для определения неизвестного? Как мож- но разделить условия на подходящие части? Подсказки 10 10.1. В чем состоит вопрос? Что имеется в виду? На самом деле, это вопрос интерпретации: вам следует рассмат- ривать слова "ровный", "восток", "запад", "север" и "юг" на идеализированном абсолютно сферическом глобусе. 10.2. Возможно ли удовлетворить условию? Является ли условие достаточным для определения неизвестных? Как мож- но было бы разбить условия на подходящие части? 10.3. Отделите различные части условия. Можете Вы за- писать их? Между стартом и местом, где три друга встреча- ются опять имеются три различные фазы: A) Боб едет с Паулем B) Боб едет один C) Боб едет с Петером
40 Пусть ?i, ^2 и t$ длительности этих фаз соответственно. Как можно разделить условие на подходящие части? 10.4. Вы знаете подходящую теорему? Основание высоты является срединной точкой основания в равнобедренном тре- угольнике. Эта теорема и ее доказательство должны быть знакомы Вам. Может, Вы используете ее метод7. Теорема по равнобедренному треугольнику доказывается из конгруэнт- ности прямоугольных треугольников, у которых высота явля- ется общей стороной. Подсказки 11 11.1. Можете Вы представить себе более доступную род- ственную задачу? Более общую задачу? 11.2. Можете Вы представить себе более доступную род- ственную задачу? Более общую задачу? Аналогичную задачу? Есть очень простая аналогичная задача: сколькими способа- ми Вы можете заплатить один цент? Есть более общая задача: сколькими способами Вы можете заплатить сумму в центов, используя центы, пятицентовые монеты, десятицентовые мо- неты, четверть- и полдолларовые монеты? Эта задача может быть решена проверкой простых частных случаев, как показа- но в короткой таблице ниже (где Еп означает количество раз- личных способов уплаты суммы в центов, использующих пять видов монет) п 4 5 9 10 14 15 19 20 24 Еп 122446699 Мы, главным образом, интересуемся ?25 и ^50, но наш вопрос, который является общим (чтобы посчитать Еп для общего п) - все еще "выделен". Есть очень простая аналогичная зада- ча: найти Лп, количество способов уплаты суммы в п центов используя только центы. (Ап = 1). 11.3. Нарисуйте чертеж. Введите удобные обозначения. Как можно получить выражение для s? Методом Евклида?
41 Методом аналитической геометрии? Тригонометрией? Какой представляется наиболее подходящим? 11.4. Введите удобные обозначения. Каково условие? Как доля каждого человека связана с долей его соседей? А с долей его соседа слева? Кто получает наибольшую сумму денег? Подсказки 12 12.1. Что неизвестно? Каковы исходные данные? Каково условие? 12.2. Что является неизвестным? Площадь D треугольни- ка. Как ее найти? Плошадь треугольника может быть вычис- лена по формуле Герона, если известны три стороны. Пусть а, бис означают длины сторон и пусть s = (а + Ъ + с)/2; тогда D2 = s(s-a)(s- b)(s-c). Стороны а, 6 и с являются сторонами прямоугольных треуголь- ников, чьи площади равны Л, В и С соответственно. Пусть стороны этих треугольников имеют длину р, q и г, так что Но площади Л, Б и С равны Л = ~qr, В = -гр, С = -pq. Мы имеем семь неизвестных D, а, 6, с, р, q, r и систему из семи уравнений для их определения. Все еще имеется препятствие: решение этой системы встречает значительные затруднения, и формула Герона может выглядеть не слишком привлекатель- ной. Попробуем начать сначала. Что является неизвестной? Площадь D треугольника. Как ее можно определить? Наиболее обычный способ вычисления площади треугольника есть ah
42 где а - основание, ah- высота треугольника с площадью D. Пусть а имеет тот же смысл, что и выше, и введем h в чертеж. 12.3. Можете ли Вы представить себе более доступную родственную задачу? Подсказки 13 13.1. Что неизвестно? Пусть ж, у и z представляют собой количество детей, возраст капитана и длину корабля соответ- ственно. Мы можем представить себе задачу так: неизвестной явля- ется тройка (ж, у, z) чисел. Отделите различные части условия. Можете Вы записать их7. (A) ж, у и z положительные целые числа отличные от 1 и такие, что xyz = 32118. (B) 4 < х < у < 100. Какой пункт (А) или (В) является более выполнимым? 13.2. Чтобы решить такую систему, необходимо комбини- ровать уравнения каким-нибудь способом. Заметим, что лю- бая перестановка ж, у, и и v оставляет систему в ее левой части неизменной. Эта симметрия подсказывает, что нам следует преобразовывать все четыре уравнения симметрично. 13.3. Можете Вы представить себе более доступную род- ственную задачу? Более обшую задачу? Что общего во всех трех суждениях? Не могли бы Вы сделать обобщение? Каждое суждение приводит к неравенству. Знаете ли Вы теорему, которая могла бы быть полезной? 13.4. Использование метода индукции требует наблюда- тельности. Видите ли Вы закономерность в рассматриваемых случаях?
43 Подсказки 14 14.1. Что является неизвестным7. Каковы исходные дан- ные*! Каковы условия! Можете Вы написать уравнение, кото- рое выражает часть условий? Вспомните, что уравнение вы- ражает ту же величину двумя различными способами. 14.2. Нарисуйте чертеж. Каково соотношение между про- центом плоскости, покрытой монетами, и процентом каждого равного квадрата или равного равностороннего треугольника, покрытого монетами? 14.3. Что такое гипотеза? Что такое умозаключение? Ги- потезой является то, что п есть целое число большее 1. Умоза- ключением является то, что пп~1 — 1 = (п — 1JР(п), где Р(п) означает целое число, зависящее от п. 14.4. Нарисуйте чертеж. Введите подходящие обозначе- ния. Как можно показать, что линейные отрезки находятся в отношении 1:2? Подсказки 15 15.1. Что является неизвестным? Каковы исходные дан- ные? Каково условие! 15.2. Можете Вы представить себе более доступную род- ственную задачу? Более общую задачу? Есть более обшая за- дача: точка Р лежит внутри прямоугольника, ее расстояния от четырех углов равны а, 6, с и d в круговом порядке (по часовой стрелке). Найдите d через а, Ь и с. 15.3. Вы знаете подходящую теорему? Попытайтесь при- думать более простое аналогичное тождество.
44 15.4. Что является неизвестным? Неизвестным является отношение объема правильного октаэдра к объему правильного тетраэдра. Что общего имеют октаэдр и тетраэдр? Jfx грани конгруэнтны равносторонним треугольникам; их ребра равны. Можете ли Вы переформулировать задачу? Вычислите объем правильного октаэдра и объем правильного тетраэдра с данной длиной ребра. Нарисуйте чертеж. Введите удобные обозначения. Решение задачи в стереометрии часто зависит от хорошего чертежа, ко- торый открывает дверь к важным соотношениям. Подсказки 16 16.1. Чтобы решить такую систему, мы вынуждены каким- нибудь образом комбинировать уравнения. Левые части сис- темы симметричны относительно ж, у и z, т.е. остаются неиз- менными при любых перестановках х,рг. Это подсказывает, что нам следует преобразовывать все три уравнения симмет- рично. 16.2. Очевидно, количество возможностей ограничено с са- мого начала D! = 24). Если Вы находчивы, Вам не надо про- верять все зти случаи. Отделите различные части условия. Можете ли Вы записать их? Пусть 6, g, w и s означают коли- чество бутылок, употребленных женами Брауна, Грина, Вайта и Смита соответственно. 16.3. Чтобы решить эту систему, необходимо каким-либо способом скомбинировать уравнения. Симметрия трех левых частей уравнений системы подсказывает, что все три урав- нения симметричны. Естественным первым шагом является исключение произведения xyz в правой части системы путем деления, но это требует исследования случаев, в которых xyz равно нулю. 16.4. Нарисуйте чертеж. Введите удобные обозначения. Решение задачи в стереометрии часто зависит от удачного чер- тежа.
45 Подсказки 17 17.1. Заметьте, что система симметрична по отношению к двум неизвестным хну. Эта симметрия должна быть как- нибудь отражена в решении. 17.2. Заметьте, что набор из четырех произведений явля- ется симметричным по отношению к а, 6, с и d. Эта симметрия должна быть как-нибудь отражена в решении. 17.3. Нарисуйте чертеж. Введите подходящие обозначе- ния. Обозначим вершины треугольника Л, Б и С, где С яв- ляется вершиной прямого угла. Пусть А', В' и С являются центрами квадратов, построенных на сторонах ВС, С А и АВ соответственно. На чертеже С оказалось лежащим на А'В1. Видите ли Вы здесь какую-нибудь пользу для доказательства этого предположения? 17.4. Нарисуйте чертеж. Введите удобные обозначения. Решение задачи в стереометрии очень часто зависит от хоро- шего чертежа. Подсказки 18 18.1. Вернитесь к определениям. Какова вписанная в тре- угольник окружность? Знаете ли Вы теорему, которая мо- жет быть полезной? 18.2. Что такое умозаключение? Выражение должно де- литься на 360. Еше выражение имеет необычную черту: его три сомножителя могут сами быть представлены как сомно- жители, чтобы образовать произведение шести сомножителей. 18.3. Заметьте, что система симметрична по отношению к трем неизвестным ж, у и z. Эту симметрию следовало бы как-нибудь отразить в решении. 18.4. Поскольку два тела имеют одинаковое количество
46 граней и одинаковый объем, попытайтесь установить, какой из них имеет меньшую плошадь поверхности. Правильность Вашего результата менее важна, чем путь, которым Вы прове- рите это. Подсказки 19 19.1. Что известно? Отношение высоты прямой призмы к стороне ее квадратного основания. Как можно определить это отношение? Если Вы вычислите выражения для двух величин, Вы можетн взять их отношение. Каковы эти величины? Как можно получить выражения для них? 19.2. Рассмотрите общий случай: каков тг-ый член последо- вательности? Можете Вы установить, что у него может быть положительный квадратный корень? Как Вы могли бы прове- рить это? 19.3. Поскольку Вы вольны выбирать собственный при- мер в каждом случае, Вы могли бы выбрать треугольник, чья площадь легко может быть вычислена. 19.4. Каковы различные возможности для сдачи или прова- ла трех предметов? Можете ли вы придумать некоторую мето- дику подсчета того, сколько студентав провалилось в каждой категории? 19.5. Нарисуйте чертеж. Введите удобные обозначения. Биссектриса d делит сторону с на два отрезка, чьи длины мы обозначим через р и q соответственно; первый имеет общий конец с а, второй - с Ь. Теперь имеем три данных: а, Ь и с; три неизвестных: р, q и d; три треугольника: первоначальный треугольник и два тре- угольника поменьше, из которого они получены разделением посредством d.
47 Подсказки 20 20.1. Разделите разные части условия. Можете Вы запи- сать их! Мы можем отметить четыре части в условии отно- сительно A) произведения лет мальчиков, B) суммы их лет, C) неопределенности задачи, D) факта (чье открытие может пройти незамеченным), что один из мальчиков является старшим. 20.2. Что известно? Площадь шестиугольника. Как ее можно определить? Является ли условие достаточным, чтобы определить неизвестное? 20.3. Что является неизвестным? Есть пять неизвестных: набор точек (А) и четыре его поднабора. Как можно охарак- теризовать такой набор? Путем установления координат ж, у точек, принадлежащих ему - путем соотношений между этими координатами! 20.4. Что неизвестно? Остаток после деления одного по- линома на другой. Можете ли Вы переформулировать задачу? Обозначим частное как q(x), а остаток как г(х). Мы должны найти полином г(х) порядка не выше 2 такой, что х + х9 + х25 + х49 + х81 = q(x)(x3 -x) + г(х).
48 Часть четвертая Решения задач Замечание. Ссылки в квадратных скобках в конце решения каждой задачи означают номера книг или статей, в которых соответствующая задача появилась прежде (во многих случа- ях с решением данным здесь или очень похожим). Статья из журнала American Mathematical Monthly, содержащая полный набор задач [24], в эти ссылки не включена. Номера страниц указывают на место расположения задачи внутри ссылки. Обо- значение "ср." (от слова "сравни") указывает на отличную, но родственную задачу или на задачи или на решения, несколько отличающихся от приведенных в этой книге. Заинтересован- ный читатель найдет решения, подсказки или комментарии в [9 - 21]. Список литературы, на которые даются ссылки, при- веден в конце книги. Решения 1 1.1. Обозначим требуемое количество пар 2п игроков как Рп. Если Вы участник, Вы можете сочетаться в пару с лю- бым, одним из 2га - 1 игроков. Раз Ваш противник выбран, то
49 остается 2n-2 = 2(n- 1) игроков, которые могут подобрать себе пару Pn-i способами. Следовательно Рп = Bп-1)Р„_1. [2; 18, с. 118]. 1.2. Плоскость, проходящая через одно ребро длиной а и перпендикуляр длиной 6, разбивает тетраэдр на два более удоб- ных конгруэнтных тетраэдра, каждый с основанием ab/2 и вы- сотой а/2. Следовательно, требуемый объем равен 1 ab a a?b ' 3 ' Т " 2 = ~6~' [2; 17, с. 235; ср. 20, с. 109 - ПО, задача 4.17]. 1.3. (A) Утверждения I и IV, вообще говоря, истинны, но утверж- дения II и III ложны; (B) Утверждения II и III ложны; прямоугольник и ромб яв- ляются контрпримерами соответственно. Утверждения II и III истинны для пятиугольника и следуют из утверждения II' и III' соответственно: II'. Если многоугольник, вписанный в окружность, явля- ется равноугольным, любые две стороны, разделенные точно одной лежащей между ними стороной, являются равными. III'. Если многоугольник, описанный около окружности, яв- ляется равносторонним, любые два угла, разделенные точно одним лежащим между ними углом, являются равными. Чтобы доказать утверждения I, II', III' и IV, соединим центр окружности с вершинами многоугольника, проведем перпен- дикуляры от центра к сторонам и выберем конгруэнтные тре- угольники. [2; ср. 19, с. 161 - 162, задача 4].
50 Решения 2 2.1. Том с 999 страницами требует 9 + 2 х 90 + 3 х 900 = 2889 цифр. Если объемистый том в вопросе имеет х страниц 189 + 3(z-99) = 1890, х = 666. [3; ср. 17, с. 234, задача 4]. 2.2. Если -679- делится на 72, оно делится как на 8, так и на 9. Если оно делится на 8, сумма 79- должна делиться на 8 (поскольку 1000 делится на 8) и таким образом 79- должна быть 792: последняя старшая цифра есть 2. Если -6792 де- лится на 9, сумма ее цифр должна делиться на 9 (правило де- ления на 9) и таким образом первая стершаяся цифра должна быть 3. Стоимость одной индюшки была (во времена дедушки) 367,92 Ч- 72 = 5,11 долларов. [3; 17, с. 234]. 2.3. Если а и —а корни с наименьшим абсолютным значе- нием, прогрессия будет иметь вид —За, , —а, а, За. Следовательно (х2 - а2) {х2 - 9а2) = ж4 - (Зт + 2)х2 + т2. Сравнивая коэффициенты при одинаковых степенях получаем систему 10а2 = Зт + 2, 9а4 = т2.
51 Исключение а дает 19т2 - 108т - 36 = 0. Следовательно, m = 6 или -6/19. [3; 17, с. 236]. 2.4. Обозначим три тождества как (а), (Ь) и (с) соответ- ственно. Если а + /3 + у = п, то (тг - 2а) + (тг - Щ + (тг - 27) = тг. Мы можем идти от (а) к F), а также от F) к (с) посредством подстановки тг — 2а, тг — 2/3 и тг — 27 вместо а, /3 и 7 соответ- ственно. Остается подтвердить (а), что может быть сделано многими способами. Например, подставим 2w, 2v и тг - 2и - 2v вместо а, /? и 7 соответственно. Тогда (а) становится sin и cos w + sin v cos и = [2 cos и cos и — cos(u + v)] sin(u + v). Используйте теоремы сложения косинуса и синуса. [3; 19, с. 162]. Решения 3 3.1. Общий закон есть (п2 + 1) + (п2 + 2) + • • • + (п + IJ = п3 + (п + IK. Члены слева образуют арифметическую прогрессию. [4; 18, с. 8]. 3.2. Условие легко разбивается на четыре части, выражен- ных четырьмя уравнениями a-d + bg'1 = 85, а + 6 = 76, а + d + bg — 84, За = 126.
52 Последнее уравнение дает а — 42, тогда второе b — 34. Скла- дывая оставшиеся два уравнения, получаем 2а -j- b(g~ + д) = 169. Поскольку а и 6 уже известны получаем квадратное уравнение относительно д. Оно дает 9 = Прогрессии = 2, d = таковы 68, 42 17, 34. -26 , 16 , 68 или или 9 17, 68, 1 ~ 2' 42, 34, d- 67 17. [4; 17, с. 236]. 3.3. Три части условия выражаются посредством Sin О; = —, а sin/3= -, о = а2+62- Исключение а к b дает х2 = с2 sin2 о; sin2 /3 sin a -j- sin /3 — 2 sin o; sin E cos 7 [4; 17, с. 237]. 3.4. В тетраэдре высота является одной стороной прямо- угольного треугольника, чья гипотенуза является ребром. Ес- ли ребро имеет длину а другая сторона треугольника имеет длину а/л/3 (это две трети от высоты грани). Поскольку дли- на высоты равна ri -f- Г2 получаем + г2 = ау/Е
53 Центр тетраэдра лежит на высоте и линейный отрезок длины г2, соединяющий центр с противоположной вершиной прямо- угольного треугольника, является гипотенузой второго прямо- угольного треугольника, чьи стороны имеют длину г\ и а/л/3. Следовательно Г2 _ 2 _ а Разделив на предыдущее равенство, получаем систему г,-п = чье решение равно г\ — ал/б/12 и г2 = ал/б/4. Следовательно П : г2 = 1 : 3. В кубе, если b является длиной ребра, то г2 = 6/2 и гз = Ьл/З/2. Следовательно г2 : г3 = 1 : \/3 и ri : г2 : гз = 1 : 3 :ЗуД. [4; ср. 18, с. 203, пример 17]. Решения 4 4.1. Если 5 есть число в последовательности, 5 должно иметь вид 11 + 100т = 4B5т + 2) + 3, где т ~ неотрицательное целое число и поэтому s оставляет остаток 3, когда делится на 4. Но квадраты имеют вид 4п2 или 4n2 -j- 4n -j- 1 и, следовательно, оставляют остатки либо 0, либо 1, когда делятся на 4. [5; 21, с. 192]. 4.2. (А) Значениям / от 1 до 9 включительно соответствуют 1, 2, 3, 4, 5, 4, 3, 2, 1 треугольников соответственно или 52 = 25 всех треугольников.
54 (В) Общий закон должен быть или смотря по тому, четно или нечетно п. Единый закон для обоих случаев: целое число, ближайшее к (п -j-1J/4. [5; ср. 18, с. 9, пример 5]. 4.3. (A) Пусть а будет стороной равностороннего треугольника. Соединяя точку внутри треугольника с тремя вершинами, Вы разбиваете его на три треугольника с площадями, которые бу- дучи сложенными вместе дают полную площадь: ах/2-\-ау/2 + az/2 = ah/2. Разделите на а/2. (B) Точка внутри правильного тетраэдра с высотой h распо- ложена на расстояниях ж, у, z и w от четырех граней соответ- ственно. Тогда x + y + z + w = h. Доказательство аналогично: разделить правильный тетраэдр на четыре тетраэдра. (C) Соотношение остается в силе в обоих случаях (А) и (В) для внешней точки, при условии, если расстояния ж, у, z (и w) берутся с соответствующим знаком: плюс (-J-), когда наблюда- тель, расположенный в точке, видит сторону (грань) изнутри, минус (—), когда он видит ее снаружи. Даказательство, в сущ- ности, то же самое. [5; 19, с. 161]. Решения 5 5.1. Общий закон есть 1-4 + 9- 16 Н + (-IO1?*2 = (-1)п"х 2 Шаг от п к п + 1 требует от нас проверить, что (-!)-(» + IJ = (-!)*(" +В(" + 2> _ (-!)-!."(
55 [6; 18, с. 116]. 5.2. При любом расположении две стороны угла должны проходить через две вершины квадрата. Пока они проходят через эту пару вершин, вершина угла движется вдоль той же круговой цуги (по теореме упомянутой в указании). Следова- тельно, каждое из двух геометрических мест точек состоит из нескольких круговых дуг: из 4 полуокружностей в случае (А) и 8 четверть окружностей в случае (В). (См. рисунок в [17, с. 244]). [6; ср. 17, с. 234 - 235, задача 7]. 5.3. 13 осей могут быть классифицированы следующим об- разом: A) 4 оси, каждая проходит через две противоположные вер- шины; углы 120°, 240° . B) 6 осей, каждая проходит через средние точки двух про- тивоположных ребер; угол 180° . C) 3 оси, каждая проходит через центр двух противополож- ных граней; углы 90°, 180°, 270°. Длины равны л/3, л/2 и 1 соответственно и средняя равна 13 [6; ср. 17, с. 235, задача 8]. Решения 6 6.1. Пусть а, Ь и с означают стороны, последняя - гипоте- нузу. Три части условия выражаются посредством а + Ъ+с = 60, а2+Ь2 = с2, аЬ = 12с. Замечая, что (а + ЪJ = а2 + 62
56 получаем F0 -сJ = с2 + 24с. Следовательно, с = 25 и либо а = 15, 6 = 20, либо а = 20, 6 = 15 (без разницы для треугольника). [7; 17, с. 237]. 6.2. Четырехугольник должен быть выпуклым. Назовем I, II, III, IV треугольники, которые получены разделением с по- мощью диагоналей; (I), (II), (III), (IV) - площади четырех тре- угольников соответственно; и р, q, г, 5 длины четырех линей- ных отрезков между вершинами и точкой пересечения диаго- налей. Введем такой порядок, при котором сторона длиной р является общей для IV и I, q - для I и II, г - для II и HI, s - для III и IV; I - противоположен III, II - противоположен IV; р + г есть длина одной диагонали, q-\-s есть длина другой диагонали. Пусть р и q образуют угол а. Тогда 2A) = pqsin a, 2A1) = 2A11) = rssina:, 2(IV) = sp sin a. Следовательно, (A) (I)(III)=(II)(IV); (B) Основание в I параллельно основанию в III, тогда и толь- ко тогда, когда p/q=r/s или (II) = (IV); (C) Четырехугольник есть параллелограмм тогда и только тогда, когда р = г, q = s или (I) = (И) = (III) = (IV). [7; 19, с. 161]. 6.3. Пусть h - общая высота, а а и 6 - радиусы нижнего и верхнего оснований усеченного конуса соответственно. Тогда радиус цилиндра равен (а + Ь)/2. Разность объемов усеченного конуса и цилиндра Ща-ЬJ
57 положительна, если только а не равно 6 и тела не совпадают. [7; ср. 20, с. 51, пример 2.54]. Решения 7 7.1. Поскольку А + В -\- С = 180°, доказательство того, что А < (В + С)/2 равносильно доказательству того, что А < A80° - А)/2 или А < 60°. Но А < 60° эквивалентно тому, что cos Л > 1/2, что наводит на мысль использовать теорему косинусов. Примем, что 6 + с > 2а. Возводя в квадрат обе стороны и применяя теорему косинусов, получим Ь2 + 26с + с2 > 4F2 + с2 - 26с cos A) или 86с cos А > 362 + Зс2 - 26с. Вычитая 46с от обеих частей неравенства, получаем 46cBcos А - 1) > 3F - сJ > 0. Следовательно, cos Л > 1/2. 7.2. Согласно четырем предположениям, объем усеченного конуса равен соответственно II. [(a2 + b2)/2]h, III. [а2 + 62 + (а + 6 IV. [а2 + Ь2 + ab]h/Z. Если 6 = а, усеченный конус становится призмой с объемом a2h: все четыре предположения находятся в согласии с полу- ченным правильным результатом. Если 6 = 0, усеченный ко- нус становится пирамидой с объемом a2h/3: только IV дает это и поэтому другие должны, быть неверными.
58 Чтобы доказать, что IV, вообще говоря, правильно, обозна- чим через х высоту маленькой пирамиды, отрезанной от пол- ной пирамиды, чтобы сделать ее усеченной. Если объемы усе- ченного конуса, полной пирамиды и маленькой пирамиды рав- ны F, А и В соответственно, то ,_, a2(x + h) Ь2х , о, . о ,2> Л F = А - В = 1 з ; - — = [a2h + (а2 - Ь2)х]-. Часть плоскости фигуры, проходящей через высоту и парал- лельную одной стороне основания, содержит подобные треуголь- ники, чьи стороны приводят к пропорции х Ь х + h a так, что bh х — а — b Подставляя это значение х в выражение для F, получаем _ Го, <22~62tIll ro to ,1^ F= \a2h + -bh\ - = [a2+ Ь2 + аЬ]-. I а — о ] 5 3 [19, с. 162; ср. 17, с. 210 - 211 и 21, с. 2 - 7]. 7.3. Допустим, что х, у и z целые. Пусть 2fc, к > 0 является наибольшей степенью двойки, которая делит ж, у и z так, что х = 2кх\ у = 2ку' и z = 2kz''. Тогда, подставляя в данное уравнение и деля на 22fc, получаем (х1J + (у'J + {z'J = 2*+VyV. Поскольку член в правой части является четным, то и левая сторона - четна, и либо х', уг и zf все четные* либо только одно из них. Но если х'', у' и z' все не равны нулю (а если один равен нулю, то и другие равны нулю), они не могут все быть четными, потому что 2 не является их общим множителем. Допустим, что х' - четное, а у' и z' - нечетные. Вычитая (х1J от обеих частей вышенаписанного уравнения, получаем
59 Как (у'J так и (z1J представляются в виде An2 + 4n + 1 и поэтому левая часть, деленная на 4, оставляет в остатке 2, тогда как правая часть делится на 4 без остатка (как х' так и число в круглых скобках - четные): противоречие. Решения 8 8.1. (А) Наименьшее возможное количество долларов в кармане равно, очевидно, 0. Следующее большее число равно, по край- ней мере, 1, следующее большее - по крайней мере, 2, ..., и количество в последнем (десятом) кармане равно, по крайней мере, 9. Следовательно, количество требуемых долларов равно, по крайней мере, Боб не может выполнить свое желание: у него только 44 дол- лара. (В) В обшем случае, залача имеет решение, если Решения нет, если - 2 [8; 9, ср. 17, с. 234, задача 3]. 8.2. Общий закон имеет вид п + + + 2! + з! + +(„+! Шаг от п к п + 1 требует, чтобы п + 1 1 (п + 2)! (п +
60 или п + 2 _ 1 (тг + 2)!~ (п + 1)!1 которое истинно для п = 1,2,3, [8; 9; 17, с. 237]. 8.3. Заметим, что первое и четвертое уравнения находятся в таком же соотношении, как второе и третье уравнения: ле- вые части имеют те же коэффициенты, но в противоположном порядке, а правые части противоположны. Складывая первое уравнение с четвертым и второе с третьим, получаем v) = 0, ) и) = 0, откуда следует, что 1 + и = 0и1/ + у = 0. Подставляя —х вместо и и —у вместо и в первоначальные уравнения, находим -4х + 4у = 16, 6х-2у = -16. Следовательно, х = —2, у = 2, и = 2, v = —2. [8; 9; 17, с. 235- 236]. 8.4. (А) Составляя уравнения, мы считаем, что х и углы а, /3, 7, S заданы, а / - неизвестна. Из AUVG находим GV через х, а + /3 и 7 (теорема синусов). Из AVUH находим HV через х,0иу + 5 (теорема синусов).Из AGHV находим / через GV, HV и S (теорема синусов), а используя выражения для GV и получаем l'2 = x2 sin2 /3 2 sin (a; -j- /3) sin /3 cos S ~sin(a + /? + 7) sin(/? + 7 + < (В) В частном случае, в котором о; = 6, E = 7, а+/? = 7+^ = 7г/2 верхнее уравнение дает х = /, как это и должно быть. Можно рассмотреть также случай, в котором а — 6, C = у,
61 но значение а + /?, не устанавливается, и случай, в котором 5, у, {3 и а подставляются вместо а, /3, у и S соответственно. Окончательно, можете также проверить размерность (см. [17, с. 202-205]). (С) Неизвестное и одно из данных меняются ролями. [8; 9; ср. 20, с. 49, пример 2.35]. Решения 9 9.1. На п-ой строчке правая часть, по-видимому, будет п3, а левая часть будет суммой п членов. Конечный член этой суммы равен нечетному числу т или 2т — 1, где п(п+ 1) Следовательно, последний член суммы в левой части должен быть 2т ~ 1 = п" -\- п — 1. Начальный член рассматриваемой суммы может быть установ- лен двуми способами: сперва путем возвращения назад на п — 1 шагов от конечного члена, затем путем продвижения на один шаг от конечного члена предыдущей строки: (п2 + п - 1) - 2(п - 1) = п2 - п + 1, ((п - IJ + (п - 1) - 1) + 2 = п2 - п + 1. Но (п2 -n+l) + (n2 -n-f 3)H |-(n2+n-l) является суммой п последовательных членов арифметической прогрессии, чья общая разность равна 2. Ее сумма равна з (Быстрая проверка убеждает, что те-ый ряд состоит из членов, среднее которых равно п2). [10; 17, с. 237].
62 0.2. Периметр шестиугольника состоит из 6п граничных линий длиной 1 и содержит вп вершин. Следовательно, V = 1 + 6A + 2: + 3 + • • • + п) = Зп2 + Зп + 1. Посредством 3 диагоналей, проходящих через его центр, шес- тиугольник разбивается на 6 (больших) равносторонних тре- угольников. Проверкой одного из них убеждаемся Т = 6A + 3 + 5 + • • • + 2гс - 1) = 6п2. Т треугольников имеют совместно ЗГ сторон. В этом общем числе ЗГ каждая внутренняя линия разделения длиной 1 счи- тается дважды, тогда как 6п линий вдоль периметра шести- угольника считаются только один раз. Следовательно, (Как следует из теоремы Эйлера для многоугольников T + V = 1 + 1). [10; 17, с. 237-238]. 9.3. Раскрывая правую часть предполагаемого тождества и приравнивая соответствующие коэффициенты, получаем аА = ЬВ = сС=1, A) ЪС + сВ = сА + аС = аВ + ЪА = 0. B) Из B) устанавливаем, что ЬС = -сВ, сА = -аС, аВ = -ЬА и перемножая эти три выражения, получаем, что abcABC = -abcABC, откуда abcABC = 0. Также из A) устанавливаем, что abcABC = 1. Следовательно, предполагаемое тождество невозможно. [10; 20, с. 54].
63 Решения 10 10.1. Требуемый кусок земли ограничен двумя меридиа- нами и двумя параллельными окружностями. Поскольку дуга параллельной окружности, отделенная двумя неподвижными меридианами, постоянно укорачивается по мере удаления от экватора, центр земли, которую хочет Боб, должен лежать на экваторе; он не сможет получить его в США. [11; 17, с. 234]. 10.2. (A) Сравнивая коэффициенты у одинаковых степеней в обе- их частях тождества, получаем 1=р2, 4 = 2pq, -2 = g2 + 2pr, -12 = 2qr, 9 = г2. Первые три уравнения, используемые последовательно, опре- деляют две системы решений р=1, q = 2, г =-3 и р=-1, q = -2, г = 3, обе из которых также удовлетворяют оставшимся двум урав- нениям. (B) Обычно невозможно удовлетворить системе с большим числом уравнений, чем число неизвестных. [11; 20, с. 54]. 10.3. (А) Боб, Пауль и Петер проделали равный путь, поэтому имеем cti - ct2 + ctz = cti + pt2 + ph = Ph + Ph + ct3. Второе уравнение дает (c-p)ti = (c-p)t3. Поскольку мы полагаем, что с > р, то t\ = ?3? т.е. Петер идет пешком столько же, сколько и Пауль. Из первого уравнения находим, что (c-p)t3 = (c + p)t2
64 и поэтому получаем Следовательно, продвижение за час равно c(h -t2-\-tz) _ с(с+Щ h+t2+tz ~ Зс + р 12 (С) В экстремальном случае р = О, (А) дает с/3, а (В) да- ет 1/3. Если р — с (А) дает с, а (В) дает 0. Эти значения интуитивно разумны. [11; 17, с. 236; ср. 20, с. 53 - 54, пример 2.60 - 2.61]. 10.4. Основание пирамиды является многоугольником с п сторонами. В случае (А) п боковых ребер пирамиды равны; в случае (В) высоты (проведенные из верхушки) ее п боковых граней равны. Проведите высоту пирамиды и соедините ее ос- нование с п вершинами основания пирамиды в случае (А), но с основаниями высот п боковых граней в случае (В). В обоих слу- чаях получаем п конгруэнтных прямоугольных треугольника. Они имеют одну сторону (высоту пирамиды) общую, а гипо- тенуза - боковое ребро в случае (А) и боковая высота в случае (В) - одной и той же длины в каждом. Следовательно, третьи стороны в конгруэнтных треугольниках должны быть равны. Поскольку они проведены из той же самой точки (основания высоты) в той же плоскости (в основании), они образуют п радиусов круга, который описан около или вписан в основание пирамиды, в случаях (А) и (В) соответственно. В случае (В) остается показать, что п упомянутых радиусов перпендику- лярны соответствующим сторонам основания, но это следует из хорошо известной теоремы стереометрии. [11; 17, с. 235].
65 Решения 11 11.1. Любая плоская фигура с центром симметрии разби- вается прямыми линиями, проходящими через этот центр, на две конгруэнтные части, следовательно, две части равной пло- щади. Требуемая линия проходит через центр симметрии. [12; 17, с. 234]. 11.2. Обозначим количество способов уплатить сумму в п центов как: Ап, если используются только центы, Вп, если используются центы и пятицентовые монеты, Сп, если используются центы, пятицентовые и десятицен- товые монеты, Dn если используются центы, пятицентовые, десятиценто- вые и четверть долларовые монеты, Еп если используются центы, пятицентовые, десятиценто- вые, четверть- и полдолларовые монеты. (Вы сможете увидеть сейчас причину для обозначения Еп). Рассмотрим случай Сп. Если десятицентовые монеты не ис- пользуются, количество способов равно Вп. Если по крайней мере используется одна десятицентовая монета, п — 10 центов остаются для уплаты в центах, пятицентовых и десятиценто- вых монетах. Следовательно, Сп = Вп + Сп-ю- Аналогично Вп — Ап + Вп_5, Dn = Сп + Dn-25i Еп — Dn + Е'п-бо Эти формулы остаются в силе, если положим Ло = Во = Со = Do = Eq = 1 и будем считать любую из величин АП1 Вп, ..., Еп, равной нулю для отрицательных п. Формулы позволяют подсчитать 3 — 4634
66 величины рекурсивно, т.е. посредством возвращения к более низким значениям п или к предшествующим буквам алфавита. Это дает следующую таблицу, которая среди прочих содержит значения для Е2ь{= ?>25) и Е50- п Ап вп Dn Еп 0 1 1 1 1 1 5 1 2 2 2 2 10 1 3 4 4 4 15 1 4 6 6 6 20 1 5 9 9 9 25 1 6 12 13 13 30 1 7 16 18 18 35 1 8 20 24 24 40 1 9 25 31 31 45 1 10 30 39 39 50 1 11 36 49 50 [12; ср. 17, с. 238, задача 20 и 20, с. 97, пример 3.84]. 11.3. Пусть а означает угол противоположный стороне а треугольника Л. Равные стороны равнобедренных треуголь- ников с основаниями бис имеют длины 6/л/З и с/\/3 соответ- ственно. Две из этих сторон образуют с s треугольник такой, что угол против s равен а + тг/3. Теорема косинусов приме- ненная к этому треугольнику, дает 3s2 = б2 + с2 - 26с cos (a + ^) . Применяя теорему косинусов к данному треугольнику Л, что- бы выразить 6сcos а и положив 6сsin а = 2Т, где Т ~ площадь Л, получим 6s2 = а2 + б2 + с2 + 4л/ЗГ. Поскольку Т симметрично относительно а, 6 и с, то это искомое выражение. [12; ср. 21, с. 26 - 28, раздел 8.4 и с.32, пример 8.3]. 11.4. Пусть Л, Б, С, D,..., J являются сидящими вокруг стола людьми, а а, 6, с, d,..., j - количество полученного ими соответственно; В сидит направо от А, С - направо от В,..., А - направо от J. Правило выражается равенствами ,_а+с _b+d д_с+е j+6 &-~' С~~' rf~~2~' •••' 2 * Первое решение. Из верхних равенств следует, что 6— а — с — Ь = d — с = — • = а — j,
67 так что доля каждого превышает долю его соседа слева на то же количество. Это постоянное превышение должно быть ну- левым, поскольку (Ь - а) + (с - Ь) + (<* - с) + • •< + (а - i) = 0. Сушествует только один способ распределить деньги: все доли равны. В.торое решение. Один (или несколько) из сидящих за столом должен получить максимальное количество. Пусть таким будет В. Тогда ни одно из количеств a, ...,j не будет больше чем b и, в частности, 6-а > 0, Ь- с> 0. Кроме того, согласно условию задачи b- a = -F- с). Следовательно, обе величины 6 — а и 6 — с должны равняться нулю. Таким образом, с также достигает максимума как d и т.д. Следовательно, а = b = с = • ¦ • = j. [12]. Решения 12 12.1. Боб имеет х марок, из которых у седьмых лежат во втором альбоме; х и у являются положительными целыми числами, 2х ух и следовательно, 3-5-7 -101 _ Х~ 28 - Знаменатель на правой стороне должен быть положительным и нечетным числом, поскольку он должен делить числитель, который является нечетным. Это оставляет три возможности:
68 у = 1, 3 и 5. Только последний случай дает делитель числите- ля. Следовательно, неизвестные определяются единственным образом: у = 5 и х = 3535. [13; 20, с. 55]. 12.2. Плоскость, проходящая через h и трех прямоуголь- ную вершину, пересекает тетраэдр, сечением которого являет- ся прямоугольный треугольник с гипотенузой /г, одной сторо- ной р и другой стороной, скажем А;, которая является высотой, перпендикулярной стороне а в треугольнике плошадью А. Сле- довательно, h2 = к2 + р2 и А = -afc. Поскольку 2D = ah, из последних двух соотношений следует, что 4D2 = a2h2 = а2(к2+р2) = 4А2 + а2р2. Используя соотношение, установленное перед этим (первона- чально данное в указаниях), получим далее AD2 = 4А2 + (г2 + q2)p2 = АА2 + (rpJ + (pqJ. Используя два соотношения, установленных ранее, и деля на 4, получим окончательное выражение которое аналогично теореме Пифагора. [13; ср. 20, с. 34, раздел 2.5B)]. 12.3. Рассмотрим прежде всего простейший специальный случай равностороннего треугольника. Симметрия может при- вести нас к предположению, что в этом случае четыре тре- угольных части будут также равносторонними. Однако, ес- ли это так, стороны треугольных частей должны быть парал- лельны сторонам данного треугольника. Это замечание ведет к конфигурации, которая решает общий случай так же, как частный случай равностороннего треугольника: посредством четырех линий, параллельных стороне данного треугольника, рассекаем каждую из двух других сторон на пять равных от- резков. Выполняя это построение три раза по отношению к
69 каждой стороне данного треугольника, мы разделим его на 25 конгруэнтных треугольников, подобных ему. Из этих 25 тре- угольников мы легко извлечем четыре треугольника, упомя- нутых в задаче; плошадь каждого из них равна 1/25 плошади данного треугольника. (Единственность этого решения не до- казана). [13; 20, с. 50]. Решения 13 13.1. Разложим 32118 на простые множители, т.е. 2 х 3 х 53 х 101. Существует только шесть способов разложить это число на произведение трех множителей, отличных от 1: 6 х 53 х 101, 2 х 101 х 159, 3 х 101 х 106, 2 х 53 х 303, 3 х 53 х 202, 2 х 3 х 5353. Только первое из этих разложений представляет собой два множителя между 4 и 100. Следовательно, у капитана 6 детей, ему 53 года и длина его корабля равна 101 футу. [14; 20, с. 139]. 13.2. Положив x + y + u + v = s (которое остается неизмен- ным при любой перестановке ar, t/, и и v) и складывая четыре уравнения, получим 8=-3 и система приводится к следующим четырем уравнениям: s — и = 4, s — х — —5, s — у = 0, s — и — —8. Следовательно, х — 2, у — —3, и = 5, v = —7. [14; ср. 20, с. 153, пример 6.24]. 13.3. Утверждение I ошибочно; утверждения II и III - вер- ны.
70 Когда три высоты, медианы или биссектрисы все лежат пол- ностью внутри треугольника (как это и есть, кроме случая вы- сот в тупом или прямоугольном треугольнике), мы имеем сле- дующую ситуацию: А, В, С являются вершинами треуголь- ника, а А', В', С - внутренние точки на противоположных сторонах соответственно; и мы должны рассмотреть сумму АЛ' + ВВ' + СС. Поскольку сумма двух любых сторон тре- угольника больше, чем третья сторона АА' + А'В > АВ, АЛ' + А'С > АС. Складывая, получаем 2АА' + ВС > АВ + СА и по аналогии 2ВВ' + СА > ВС + АВ, 2СС -{-АВ > С А + ВС. Складывая последние три неравенства, получим 2{АА' + ВВ' + СС) > АВ + ВС + С А. Следовательно, утверждения II и III верны, а утверждение I верно для острых треугольников. В качестве контрпримера к утверждению I рассмотрим рав- нобедренный треугольник с основанием b и углами при осно- вании А. Когда А стремится к 0, каждая высота стремится к нулю, в то время как периметр стремтся к 26. Угол Л, доста- точно близкий к 0, опровергает утверждение I. [14; 19, с. 214, пример 16.6.1]. 13.4. Обследованные случаи наводят на мысль, что 1!1 + 2!2+3!3-| + п\п= (п + 1)!- 1. Это может быть доказано математической индукцией или сле- дующим способом: (п 4-1)! - п\ = п\(п -f 1) - п\ = п\п.
71 Поэтому для п = 1, 2, 3, ..., п: 2!2 = 3! - 2!, 3!3 = 4! - 3!, п\п = (п + 1)! — п\ Сложение этих соотношений приводит после очевидных сокра- щений к требуемому результату. [14]. Решения 14 14.1. Используем следующие обозначения: и - скорость Эла, v - скорость Билла, t\ - время от начала до первой встречи, *2 - время от начала до второй встречи, d - длина улицы. Тогда ut\ = a, ut2 = d-\-b, vti = d — a, vt2 = 2d — b. (A) Выражая u/v двумя различными способами, получаем a d-\~b d — a 2d — b Следовательно, после приведения подобных членов находим d = За — 6. (B) Эл придет быстрее. Численно: u/v = 3/2. [20, с. 53]. 14.2. Для первой структуры 1007г/4, а для второй 100тг/Bл/5) или приблизительно 78,54% и 90,69% соответственно. Переход
72 от большого (квадратного) стола к бесконечнему, в действи- тельности, включает понятие предела, но мы не настаиваем на этом, поскольку результат интуитивен. [20, с. 50]. 14.3. Отметим прежде, что если п больше 1, частное от пп - 1 и п - 1 равно пп-2 _|_пп-3 + ...+ п+1< Можно понять эту сумму, как получающуюся из полинома R(x) = A + х)п-2 + A + х)п~3 + ¦ ¦ ¦ + A + х) + 1, если подставить в него п — 1 вместо х. В раскрытии R(x) по степеням х (можно использовать формулу бинома) член, неза- висимый от ж, равен #@) = п — 1 и поэтому R(x) = Qn-ъ {х)х + п - 1, где <2п-з(ж) ~ полином степени п — 3, чьи коэффициенты яв-- ляются целыми. Подставляем п — 1 вместо х и приходим к заключению: = (п - l)[Qn-3(n - 1)(п - 1) + п - 1] = 14.4. Пусть А, В и С являются вершинами данного тре- угольника и пусть а, 6 и с являются длинами противолежащих сторон соответственно. Рассмотрим сторону шестиугольника противоположную стороне длиной а и обозначим ее длину как а'. Рисуя различные фигуры и рассматривая крайние случаи, мы видим, что нам необходимо показать, что а' = 2<zm, где ат - длина медианы ААВС, проведенной к стороне длиной а. Продолжим АС к D так, что AD = b и соединим D с В. Тогда ACDB ~ ДСАА1, где А' - средняя точка стороны ВС, так что АА' — ат. Следовательно, медиана ААВС длины ат параллельна BD и ат = BD/2. Но треугольник, чьи стороны имеют длины а', 6 и с конгруэнтен AABD (по двум сторонам и углу между ними). Следовательно, ат = а'/2-
73 Решения 15 15.1. Если снижение цены равно х центов и имеется у ручек в оставшемся запасе х < 50 и ху = 3193. Тогда 3193 = 31 х 103 является произведением двух простых чисел и поэтому оно имеет точно четыре различных множите- ля 1, 31, 103 и 3193. Если мы положим что х - целое, то х = 1 или 31. Если мы положим также, что х > 1, то х = 31. [15; 20, с. 55]. 15.2. Пусть расстояния от Р до четырех сторон прямо- угольника равны х, г/, xf, у' в указанном порядке. С выбран- ными обозначениями а2 = у'2 + х2, Ь2 = х2+у2, с2 = у2 + х'2, d2 = х'2 + у12, и поэтому а -\- с =о + а . В нашем случае а = 5, b — 10, с = 14 и поэтому d2 = 25 - 100 + 196 = 121, d=ll. Отметим, что данные a, b и с, которые определяют d, недоста- точны, чтобы определить стороны х + х' и у Л-у' прямоуголь- ника. [15; 20, с. 50]. 15.3. Напишем известное соотношение sin 2a = 2 sin a cos a и подставим в него последовательно а/2, а/4 и а/8 вместо а. Получим a sin a cos — = 2 2sinf
74 a Sin ту COS — = 7F 4 2sm^ a cos — = 2 sin TV Умножение трех равенств дает после очевидных сокращений требуемое тождество. Если мы последовательно продолжим зти равенства до п, то их произведение дает а он ос он sin он cos - cos - cos - •. -cos — = ——ьт. Z 4: a L ?, Sin qTJ" Можно также предположить эту более общую формулу и дока- зать ее методом математической индукции. [15]. 15.4. Пусть О означает объем октаэдра, Т - объем тетра- эдра, а а - длину ребра. Первое решение. Октаэдр разделен соответствующей плоскостью на две конгруэнтные правильные пирамиды, чье общее квадратное основание имеет площадь а2. Высота одной из этих пирамид равна а/у/2 (на хорошем чертеже она прохо- дит через диагональ основания) и поэтому Л а2 а а3^2 Проведем плоскость через высоту (длиной h) тетраэдра и че- рез боковое ребро. Сечение разделяется высотой на два прямо- угольных треугольника, из которых получаем 2 2 и поэтому Окончательно BayJ V 6 т ) 1 a t 32 2 (ауД\ V 2 ) Зау/2 = 4Т. faV3 \ 6 а3 у/2 12 ' Л 2а2 ) з
75 Второе решение. Рассмотрим правильный тетраэдр с ребром 2а; его объем равен 23Т. Четыре плоскости, каж- дая из которых проходит через средние точки трех его ребер, оканчивающихся в той же вершине, рассекают его на четы- ре правильных тетраэдра, каждый объемом Т, и правильный тетраэдр объемом О. Следовательно, AT + О = 8Г и опять приходим к О = AT. [15; 20, с. 51]. Решения 16 16.1. Простейшим выражением, которое симметрично по х, у и z, является их сумма. Складывая три предложенных уравнения, получим ЮОООя + ЮОООу 4- lOOOOz = 20000, х + у + z = 2. Вычитая 2134ж + 2134г/ + 2134z = 4268 из каждого из трех предложенных уравнений, получим три но- вых уравнения, решения которых равны х = 1, у = -1, z — 2. [16; ср. 20, с. 153, пример 6.24]. 16.2. Условие легко разбивается на две части, выраженные двумя уравнениями Ь + 2д + 3w + 45 = 30. Вычитая первое из второго, получаем g + 2w + 3s= 16,
76 которое показывает, что либо д и s оба нечетные, либо оба четные. Следовательно, есть только четыре случая, которые нуждаются в проверке: д s w = 8- (# + 3s)/2 3 5 - 1 5 3 1 2 4 1 4 2 3 Только последний случай приемлем. Следовательно, s = 2, w = 3, g = 4, Ь = Ъ и леди имеют имена: Анна Смит, Бетти Байт, Кэрол Грин, Дороти Браун. [16; 20, с. 149]. 16.3. Если х = 0, второе (или третье) уравнение дает у2z2 = 0 и поэтому еще одно из неизвестных у или х долж- но быть равно нулю. Следовательно, либо х, у и z все отличны от 0 или, наконец, два исчезают. Если любые два исчезают, уравнения удовлетворяются. Теперь рассмотрим случай, в котором ни одно из трех не- известных не равно 0. Делением получаем систему zx Ml + *3L - ь х * z — w' Ul + zx. - c x + у Складывая эти три уравнения и деля на 2, получаем yz zx xy | | = x у z z Из этого уравнения вычитаем каждое из трех уравнений пред-
77 шествуюшей системы и получаем Ul ~ -а + Ъ + X ZX У z а а 2 -b-\- '2 + 6- '-> - с С • Произведение этих трех уравнений равно xyz = (-а + b + с)(а - b + с)(а + b - с)/8. (*) Разделим его на каждое уравнение из предшествующей систе- мы и после извлечения квадратного корня получим х = [{а-Ъ + с)(а + Ь- у = [(-а + Ь + сНа + б z = [(-а + 6 + с)(а-6 Мы должны принять во внимание, однако, два значения квад- ратного корня. Сосредоточимся на вызывающем на размышление частном случае и положим, что а, 6 и с являются длинами трех сторон треугольника. Тогда из (*), xyz положительно и, следователь- но, только следующие четыре комбинации знаков допустимы: х + + - - У + - + - z + - - + [16; 21, с. 193]. 16.4. Проведем плоскость через высоту пирамиды и че- рез среднюю точку одной стороны (длиной а) ее основания. Пересечением этой плоскости с пирамидой является равнобед- ренный треугольник, который может быть использован в ка- честве ключевого чертежа: его высота равна Л, его стороны имеют длину / (где / - высота боковой грани пирамиды) и его
78 основание равно 2b (где Ь - высота одного из шести конгруэнт- ных' равносторонних треугольников, составляющих основание пирамиды). Плошадь основания равна S баб 4 = ^~' плошадь боковой поверхности равна Т = ~2~ и поэтому / = 36. Используя ключевой чертеж, получаем h2 + Ъ2 = I2 = 9Ь2 и поэтому Имеем также и поэтому Следовательно, [16; ср. 20, с. 2 «2 462 _ — _ S = Ylab = h 51, пример 2.49]. а2 h2 6 Решения 17 17.1. Нам требуется найти точки пересечения двух конгру- энтных эллипсов, симметричных друг относительно друга и по отношению к линии х = у. Вычитание уравнений дает х2 = у2.
79 Существует четыре точки пересечения: F,6), (-6, -6), B, -2), [Ср. 21, с. 162, пример 15.36]. 17.2. Произведение четырех данных произведений симмет- рично по а, 6, с и d. Если мы сможем показать, что 4аA - 6L6A - сLсA - й)Щ\ - а) < 1, то из этого будет следовать, что не все из четырех данных произведений больше, чем один. Нам дано, что 0 < а < 1, О < 6 < 1, 0<с<1и0<й<1. Рассмотрим произведение 4аA — а), которое положительно, т.к. его сомножители положи- тельны; каков его максимум? Очевидно, что максимум равен 1, который достигается при а = 1/2. Для того, чтобы прове- рить это заметим, что 1 - 4аA - а) = A - 2аJ > 1, и что равенство достигается только, когда а = 1/2. Подоб- но этому произведения 46A - 6), 4сA - с) и 4d(l - d) также положительны и не более единицы. Следовательно, 4аA - аL6A - 6LсA - с)Ы{\ -d)<\. Перегруппировывая члены, 4аA - 6L6A - сLсA - dLd(l - а) < 1. 17.3. (A) Поскольку LB'CA = lA'CB = 45° и LC = 90°, то С лежит на линейном отрезке А'В'. ААВС и ААВС являют- ся прямоугольными треугольниками; они могут быть вписаны в окружность с диаметром АВ. LACC и LBCC отделяют равные дуги окружности так, что они равны. Следовательно, LB'CC = lA'CC = 90°. (B) Если стороны треугольника имеют длины а, 6 и с, то В'С имеет длину 6/>/2, А1 С - длину а/л/2, и А'В', следова- тельно, длину (а + Ь)/л/2.
80 Пусть D является вершиной, противоположной А, в квад- рате на стороне Ъ. Тогда ААСС ~ /S.ADB (пары соответству- ющих углов равны). Следовательно, СС : АС = DB : AD и длина СС также равна (а + Ь)/\/2. (Доказательство, использующее проеобразлвания общего слу- чая, в котором треугольник не обязательно является прямо- угольным, дано в [22, с. 96 - 97]). 17.4. (А) Проведем плоскость через ребро длиной Ь и среднюю точку М противоположного ребра. Пересечением этой плос- кости с тетраэдром является равнобедренный треугольник, ко- торый может быть использован как ключевой чертеж: его ос- нование имеет длину Ь, его стороны имеют длину а%/3/2 (они являются высотами равносторонних граней тетраэдра) и, сле- довательно, его высота равна >/3«2 — Ь2/'2. Из-за симметрии, центр О описанной сферы лежит на линии, соединяющей А/ со средней точкой В основания этого треугольника. Также линия, соединяющая О с центром С одной из равносторонних граней тетраэдра, перпендикулярна этой грани. Точка С раз- деляет каждую высоту (или медиану) равносторонней грани в отношении 1 : 2. Следовательно. СМ имеет длину а\/3/б. Тре- угольник ОСМ подобен каждому треугольнику на которые ключевой чертеж разделен посредством MB. Следовательно, отношения соответствующих сторон равны. Если х является длиной ОБ, получаем >/3a2 - 2 0 b2 - x а\/3 уЪаг - 2 b2 Решая это уравнение относительно х, получаем 2а2 - Ъ2 х = a2 - Ь2 Если г - радиус описанной сферы 2_ 2 , ^ V _ Bа2 ~ Ь2J | б2 __ а2Dа2 - б2) (Ь\ _ {2а2-Ь2) V2/ ~ 4Cа2 - б2 ) + 4 ~ 4Cа2 - б2)
81 а /4а2 - Ь2 г = — 2 V За2 - б2' Заметим, что знаменатель равен нулю, когда 6 = ау/д. Это предельный случай, поскольку для тетраэдра, рассмотренного в задаче, b < ал/3. (В) Радиус сферической поверхности может быть опреде- лен посредством способа, в котором четыре точки расположе- ны так, чтобы образовать вершины двух конгруэнтных рав- носторонних треугольников, имеющих общую сторону и по- вернутых вокруг этой стороны. Пусть стороны треугольника имеют длину а. Располагая четыре точки в соприкосновении с поверхностью и измеряя расстояние Ъ между двумя точка- ми, которые не являются концевыми точками общей стороны, можно использовать результат (А), чтобы вычислить радиус сферы, определяемый четырьмя точками. [21, с. 188]. Решения 18 18.1. Каждая вершина треугольника равноотстоит от двух из трех точек, в которых стороны треугольника касательны вписанной окружности. Обозначим А, В и С вершины против сторон a, b и с соответственно. Тогда расстояния от С до двух ближайших точек касания равны d/2, расстояние от В до двух ближайших точек касания равно а — (d/2) и расстояние от Л до двух ближайших точек касания равно b— (d/2). Поскольку а - {d/2) f b - (d/2) = с, то a + b~c + d. [21, с. 188]. 18.2. Первое решение. Перепишем выражение в виде п[(п - 2)(п - 1)п(п + 1)(п + 2)]
82 и заметим, что выражение в квадратных скобках является про- изведением пяти последовательных целых чисел. Мы замеча- ем, что 360 = 23 • З2 • 5. В любой последовательности пяти сле- дующих друг за другом целых чисел одно должно быть кратно 5, следовательно, 5 делит без остатка выражение. Также одно из этих пяти целых чисел должно быть кратным 4 и посколь- ку, наконец, одно должно быть четным, 8 должно делить без остатка выражение. Если п кратно 3, то п2 кратно 9, и 9 делит без остатка выражение. Если п не кратно 3, то или п — 2 и п + 1 или 7i — l и п + 2 делится без остатка на 3 и, следовательно, 9 делит без остатка выражение. Поскольку выражение делит- ся без остатка на 5, 8 и 9 и поскольку они не имеют общих множителей, то выражение должно делиться без остатка на их произведение. Второе решение. n2(n2-l)(n2-4) _ 360 - f(n + 3) + (п ~ Шп + 2)(п + 1)п(п -!)(»- 2) _ 6! п 4- 3\ /га + 2 6 J + V 6 а биномиальные коэффициенты являются целыми числами. [Ср. 21, с. 191, пример 3.65.1]. 18.3. Вычитая по очереди второе уравнение из первого, третье из второго и первое из третьего, получим новую систе- му -5х2 4- Ay2 + z2 = О, х2 - Ъу2 4- 4г2 = О, Ах2 + у2 — bz2 = 0. Мы можем исключить z2, умножая первое уравнение в этой системе на —4 и складывая его со вторым. Мы можем исклю- чить а;2, умножая второе уравнение на -4 и складывая его с третьим. Это дает
83 Подстановка в первоначальную систему дает восемь реше- ний A,1,1), (-1,-1,-1), (о, — 3, — о), (—3,о, о), (—3, о,—3), C,-3,3), (-3,-3,3), C,3,-3). [Ср. 21, с. 163, пример 15.37- 15.38]. 18.4. Пусть г является радиусом окружности, вписанной в основание призмы. Основание состоит из шести равносторон- них треугольников, чьи стороны имеют длину 2г/>/3. Объем призмы равен Площадь поверхности призмы равна ^ 2 • 6-^= + 6-^2г = 4\/3 • г2 + SVS • г2 = л/3 УЗ г Пусть а является длиной ребра правильного октаэдра, чей объ- ем равен 4л/3 • г3. Октаэдр может быть разделен на две кон- груэнтные пирамиды, чье общее основание является квадратом со стороной длиной а и чья высота равна половине диагонали квадрата со стороной длиной а. Поэтому, объем октаэдра равен 1 9 <*> a3v/2 2 И а = \/б • г. Площадь поверхности октаэдра равна ^^ = 2УЗ • а2 = 2sfl • 6г2 = г Следовательно, площади двух поверхностей равны. Рассматривая два тела с одним и тем же числом граней и одинаковым объемом, естественно было бы ожидать, что если одно из них правильное, то оно будет иметь площадь поверх- ности меньшую, чем другое тело. [Ср. 21, с. 162, пример 15.29 - 15.30].
84 Решения 19 19.1. Пусть s является длиной стороны основания данной призмы (торта). Тогда высота данной призмы равна 5s/16, объем равен 5s3/16, а поверхность E граней) с сахарной гла- зурью равна 9s2/4. Для требуемой призмы (кусок торта) пло- щадь основания равна s2/4, а объем равен 5s^/A6 x 9). Высота равна 5s/36, частному объема и площади основания. Сторона основания требуемой призмы равна s/2. Следовательно, тре- буемое отношение равно 5s 2 _ _5_ 36 ' ~s ~ 18" На верхней поверхности торта (квадрат) отметим концентри- ческий квадрат. Стороны меньшего квадрата параллельны и равны половине длины сторон большего квадрата. Линейные отрезки соединяют соответствующие вершины и средние точ- ки соответствующих сторон. Каждый из 8 кусков с сахарной глазурью на стороне явля- ется прямой призмой, из которой вырезана более маленькая прямая призма; для обеих призм, большей и меньшей, основа- ние является равнобедренным прямоугольным треугольником. Другое решение может быть получено путем вращения квад- ратного куска около его центра на 45°. [21, с. 188]. 19.2. Первое решение. Вспомните формулу для суммы геометрической прогрессии. Сумма последовательнос- ти, имеющей цифр 2тс, равна 9 + 8A0 + 102 + • • • + Ю") + 4A0n + 10n+1 + • • • + 102n-1) = = 1 + (8 + 4 • 10n)(l + 10 + • • • + Ю7^1) = 9 • Юп 4- 1\2 Это - квадрат целого числа, поскольку B • 10п + 1)/3 = 1 + 6A0п - 1)/9 = 666 ... 67
85 число с п цифрами. Второе решение. Экспериментирование с некоторыми примерами 49 = 72, 4489 = 672, 444889 = 6672 приводит к предположению, что n-ый член имеет вид F66 .. .67J где 666 .. .67 имеет п цифр. Чтобы подтвердить предположение доказательством, можно начать с замечания, что 666...667Х6 = 4000...002, 666...667Х7 = 4666...669 и увидеть обычный пример умножения целых чисел, записан- ных десятичными арабскими цифрами. 19.3. Приведем простые примеры прямоугольных треуголь- ников: стороны площадь 3 4 5 6 1 1 у/2 1/2 >/2 уД 2 1 12 Существует много других примеров, но треугольники, которые не являются прямоугольными, менее удобны. 19.4. Первое решение. По существу, мы рас- сматриваем здесь три множества Л, В и С и их пересечения. Если каждое множество представить точками внутри круга (диаграмма Венна или более правильно круг Эйлера), то три круга, каждое из которых пересекается с двумя другими, де- лят плоскость на восемь областей. Одна из этих областей вне трех кругов - бесконечна; задача заключается в том, чтобы определить количество индивидов, принадлежащих к этой бес- конечной области. Начиная с области внутри всех трех кругов (с правого конца ряда данных чисел) и продвигаясь наружу (налево в ряду), можно посчитать, сколько индивидов в каж- дой области. Пусть X означает множество индивидов вне мно- жества X, пусть XY означает пересечение множеств X и Y и пусть [X] означает количество индивидов в множестве X.
86 Тогда количество индивидов в объединении Л, В и С дается выражением [ABC] + [ABC] + [ABC] + [ABC] + [ABC] + [ЛВС] + [ABC] = Таким образом, требуемое число (студентов сдавших все три экзамена) равно 41 - 15 = 26. Второе решение. Используем те же обозначения, что и в первом решении. Количество индивидов в объединении А, В и С также дается выражением [А] + [В] + [С] - [АВ] - [АС] - [ВС] + [ABC] Индивид, который принадлежит точно одному из трех мно- жеств, считается один раз в первой строчке и больше нигде. Индивид, который принадлежит точно двум множествам, счи- тается дважды положительно в первой строке и один раз от- рицательно во второй строке. Индивид, который принадлежит всем трем множествам, считается три раза положительно в первой строке, три раза отрицательно во второй и один раз положительно в третьей. Таким образом, предложенное выра- жение считает каждого из индивидов, которые должны быть посчитаны только один раз (и это может быть обобщено на большее число множеств). Применяя общее выражение к нашему данному случаю, на- ходим, что 12 + 5 + 8 - 2 - 6 - 3 + 1 =15 студентов провалили один или более предметов, и поэтому 41- 15 = 26 сдали все три экзамена. 19.5. (А) Наша задача значительно облегчается, если мы учили и помним классическое утверждение геометрии (Euclid VI 3):
87 отрезки основания р и q имеют то же отношение, что и смеж- ные стороны а и Ь р _ а Поскольку р + q = с, имеем ас be Р= а + 6' Применяя теорему косинусов дважды к двум различным тре- угольникам, которые оба содержат угол #, противоположный стороне а в исходном треугольнике, получим а2 = Ь2 + с2 - 2Ьс cos a, rf2 = b2 +q2 - 2bqcosa. Исключая cos а и решая относительно d2 получим 2" d2 = 1- (Если Вы не знаете или не можете вспомнить это утверждение об отношении pfq% можно найти решение применением триго- нометрических соотношений в трех треугольниках. Это сле- дует, в действительности, из теоремы синусов, примененной дважды к двум меньшим треугольникам). (В) Частный случай. Если а = 6, данный треугольник является равнобедренным с основанием с и формула дает г2 Предельный случай. Если данный треугольник вырождается, сводится к линейному отрезку с, то a -f- 6 = с и формула дает d — 0. Формула может быть также проверена на размерность [17, с. 202 - 205]. [21, с. 195].
88 Решения 20 20.1. Если Вы запишете все двенадцать разложений 72 на три множителя и обратите внимание на сумму сомножите- лей, Вы обнаружите, что единственная сумма, встречающаяся дважды, равна 14(= 2-6-6 = 3-3-8). Следовательно, номер улицы равен 14. Принимая во внимание старшего мальчика, возрасты равны 3, 3 и 8. [21, с 192]. 20.2. Пусть а и Ь означает длины сторон. Шестиугольник состоит из трех квадратов плошадью а2, б2 и с2 соответствен- но и четырех треугольников с одинаковой площадью А. Либо введите вспомогательный линейный отрезок в чертеже, чтобы доказать, что один из двух тупых треугольников имеет высо- ту а и основание 6, а другой - высоту b и основание а, либо используйте тригонометрию: А = -аЪ = -acsin/? = racsinA80° - /3). Следовательно, площадь шестиугольника равна а2 + Ь2 + с2 + АА = 2с2 + 4А. [21, с. 187]. 20.3. (A) Треугольник с вершинами A,1), @,1), A/2,1/2) и сто- ронами, расположенными на прямых линиях у = 1, х = у, причем х + у > 1; (B) Две стороны треугольника (А) на линиях у — 1 и х — у\ (C) Дуга окружности х2 + у2 — 1 внутри треутольника (А); (D) Часть треугольника (А) поверх дуги (С); (E) Часть треугольника (А) ниже дуги (С). Точка A,1) представляет равносторонний треугольник; точ- ка A/\/2,1/л/2) представляет равнобедренный прямоугольный треугольник; точка A/2, \/3/2) представляет треугольник с
89 углами 30°, 60°, 90°; и сторона треугольника (А) на линии х + у = 1 представляет вырожденные треугольники. [Ср. 21, с. 146 - 149 и с. 158, пример 15.3]. 20.4. Остаток г(х) является полиномом степени, не превы- шающей 2: г(х) — а + Ьх + сх2. Тогда х + х9 + х2Б + х49 + х81 = q{x) (ж3 - х) + а + Ьх + ся2. Это дает для а; = -1, 0 и 1 соответственно —5 = а — b + с, 0 = а, 5 = а + 6 + с. Следовательно, а = 0, 6 = 5, с = 0и требуемый остаток равен Ьх. [Ср. 21, с. 139, пример 14.24].
90 Литература 1. R. Creighton Buck. A look at mathematical competitions, American Mathematical Monthly, 1959, 66 p. 201-212. 2. Department of Mathematics, Stanford University, The Stan- ford University Mathematics Examenation, American Math- ematical Monthly, 1946, 53, p. 406-409. 3. -, Stanford University Mathematics Examenation, American Mathematical Monthly, 1947, 54, p. 430. 4. -, Stanford University Competitive Examenation, American Mathematical Monthly, 1948, 55, p. 448. 5. -, Stanford University Competitive Examenation in Mathe- matics, American Mathematical Monthly, 1949, 56, p. 496- 497. 6. -, Stanford University Competitive Examenation in Mathe- matics, American Mathematical Monthly, 1950, 57, p. 651- 652. 7. -, Stanford University Competitive Examenation in Mathe- matics, American Mathematical Monthly, 1952, 59, p. 127.
91 8. -, Stanford University Competitive Examenation in Mathe- matics, American Mathematical Monthly, 1953, 60, p. 571- 572. 9. G. Polya. The 1953 Stanford University Competitive Exa- menation: problems, solutions and 'comments, California Mathematics Council Bulletin, 1953, A), 11, p. 15-17. 10. -. The 1954 Stanford University Competitive Examenation in Mathematics: problems, solutions and comments, Califor- nia Mathematics Council Bulletin, 1954, B), 12, p. 7-8. 11. . The 1955 Stanford University Competitive Examenation in Mathematics: problems, solutions and comments, Califor- nia Mathematics Council Bulletin, 1955, B), 13, p. 15-17. 12. -. The 1956 Stanford University Competitive Examenation in Mathematics: problems, solutions and comments, Califor- nia Mathematics Council Bulletin, 1956, B), 14, p. 19-22. 13. -. The 1957 Stanford University Competitive Examenation in Mathematics, California Mathematics Council Bulletin, 1957, B), 15, p. 18-21. 1 1. . The 1958 Stanford-Sylvania Competitive Examenation in Mathematics, California Mathematics Council Bulletin, 1958, A), 16, p. 18-20. 15. . The 1960 Stanford-Sylvania Competitive Examenation in Mathematics, California Mathematics Council Bulletin, 1960, B). 18, p. 16-17. 16. -. The 1961 Stanford-Sylvania Competitive Examenation in Mathematics, California Mathematics Council Bulletin, 1961, B), 19, p. 10-11. 17. . How to Solve It, 2nd ed., Doubleday Anchor A 93, 1957, Princeton Paperback, 1971. 18. -. Mathematics and Plausible Reasoning, Vol. 1, Princeton Univ. Press, 1954. 19. -. Mathematics and Plausible Reasoning, Vol. 1, Princeton Univ. Press, 1968.
92 20. -. Mathematical Discovery, Vol. 1, Wiley, 1962. 21. -. Mathematical Discovery, Vol. 2, corrected printing, Wiley, 1968. 22. H.S.M. Coxeter and S.L. Greitzer. Geometry Revisited, Ran- dom House, 1967. 23. Hungarian Problem Book I, II, Random House, 1963. 24. G. Polya and J. Kilpatrick. The Stanford University Competi- tive Examenation in Mathematics, American Mathematical Monthly, 1973, 80, p. 627-640.
Научно-популярное издание ПОЙА Джордж КИЛПАТРИК Джереми СБОРНИК ЗАДАЧ ПО МАТЕМАТИКЕ СТЭНФОРДСКОГО УНИВЕРСИТЕТА с подсказками и решениями Лицензия ЛР№ 071872 от 24 мая 1999 г. Подписано в печать 01.12.2001. Формат 60x90 1/16 Бумага офсетная. Гарнитура «Computer Modern». Печать офсетная. Усл.Печ.л. 6. Зак. № 4634 НО Научный Фонд «Первая Исследовательская Лаборатория имени академика В.А. Мельникова» 117997 г. Москва, ГСП-7, Нахимовский проспект, 36, к. 1, комн. 816 (ИМВС РАН). e-mail: mindiaOmail.imvs.ru Отпечатано с оригинал-макета в ППП Типография «Наука» 121099, Москва, Шубинский пер., д.6. Налоговая льгота — общероссийский классификатор продукции ОК-005-93, том 2; 953000 — книги, брошюры